SlideShare a Scribd company logo
1 of 47
Download to read offline
CRITICAL
                      REASONING
                   PRACTICE SETS                                         CHAPTER 16




CRITICAL REASONING SET 1
          Time: 25 minutes—16 Questions
          Directions: For each question in this section, select the best of the answer choices given.

1. One problem with labor unions today is that their    2. Opening a plant in war-torn Country X is not
   top staffs consist of college-trained lawyers,          inadvisable, despite what critics of the plan may
   economists, and labor relations experts who             say. Ten years ago we opened our plant in
   cannot understand the concerns of real workers.         Country Y in the middle of a revolution; that
   One goal of union reform movements should be            plant has been generating substantial profits
   to build staffs out of workers who have come up         ever since.
   from the ranks of the industry involved.
                                                            Which of the following is the author of the
    The argument above depends primarily on which           argument above most reasonably intending the
    of the following assumptions?                           reader to conclude?

    (A)   Higher education lessens people’s                 (A)   Wars are profitable for the author’s
          identification with their class background.             particular business.
    (B)   Union staffs should include more people           (B)   Country X is a more politically stable
          with first-hand industrial supervisory                  nation than is Country Y.
          experience.                                       (C)   Critics of the proposed plant in Country X
    (C)   People who have worked in a given                       are likely to be biased.
          industry can understand the concerns of           (D)   The proposed plant in Country X will
          workers in that industry.                               generate profits despite the war.
    (D)   Most labor unions today do not fairly             (E)   The proposed plant in Country X will be
          represent workers’ interests.                           more successful than the plant in Country
    (E)   A goal of union reform movements should                 Y.
          be to make unions more democratic.




                                                                        GO ON TO THE NEXT PAGE

                                                                                                           177
P R A C T I C E S E T S A N D E X P L A N AT I O N S



3. Since the deregulation of the air travel industry    4. One Zydol capsule contains twice the pain
   in the late 1970’s, air fares have been relatively      reliever found in regular aspirin. A consumer will
   low and the number of passenger miles flown             have to take two aspirin in order to get the relief
   has been increasing. In recent months, however,         provided by one Zydol. And since a bottle of
   air fares have risen but the number of passenger        Zydol costs the same as a bottle of regular
   miles flown is still going up.                          aspirin, consumers can be expected to switch to
                                                           Zydol.
    Which of the following can most reasonably be
    inferred from the statements above?                     Which of the following, if true, would most
                                                            weaken the argument that consumers will be
    (A)    The cost of air travel does not affect the       discontinuing the use of regular aspirin and
           number of passenger miles flown.                 switching to Zydol?
    (B)    People are now flying to destinations that
           they formerly reached by train or by other       (A)   A regular bottle of aspirin contains more
           modes of transportation.                               than twice as many capsules as does a
                                                                  bottle of Zydol.
    (C)    Factors other than low air fares must be
           contributing to the rise in the number of        (B)   The pain reliever in Zydol is essentially the
           passenger miles flown.                                 same pain reliever found in regular aspirin.

    (D)    Takeovers in the deregulated air travel          (C)   Some headache sufferers experience a
           industry have been responsible for the                 brief period of dizziness shortly after
           rise in air fares.                                     taking Zydol but not after taking regular
                                                                  aspirin.
    (E)    Air fares can climb even higher without
           causing a drop in the number of                  (D)   Neither regular aspirin nor Zydol is as
           passenger miles flown.                                 effective in the relief of serious pain as
                                                                  are drugs available only by prescription.
                                                            (E)   A Zydol capsule is twice as large as the
                                                                  average aspirin.




                                                                        GO ON TO THE NEXT PAGE

 178
CRITICAL REASONING PRACTICE SETS



5. At a certain college, graduate teaching               6. Air travel is becoming increasingly more
   assistants conduct discussion sections but have          dangerous. In the last year there have been
   no input into grading. It has been suggested that        seven major collisions resulting in over 700
   graduate assistants be given some grading                deaths, more deaths than in any previous year.
   responsibility, but many undergraduates oppose
   that proposal. They argue that if grades are             Which statement, if true, would most weaken
   assigned by graduate assistants, regular full-           the argument above?
   time faculty will devote less time and attention
                                                            (A)   Since the volume of air traffic has been
   to undergraduate work.
                                                                  increasing all the time, an increase in the
    The information in the passage above answers                  number of deaths due to collisions does
    which one of the following questions?                         not necessarily mean greater danger.
                                                            (B)   The increase in collisions can be explained
    (A)   Are grades assigned by graduate teaching                by statistical coincidence, hijackings, and
          assistants inherently as fair as those                  unusual weather.
          given by regular faculty?
                                                            (C)   Mortality per passenger mile is lower for
    (B)   Are some undergraduates in favor of                     air travel than for any kind of surface
          maintaining the full-time faculty’s interest            transportation.
          in their schoolwork?
                                                            (D)   The increase in deaths due to collision in
    (C)   May regular full-time faculty conduct                   air travel has proceeded at a rate identical
          discussion sections at the college?                     to that for deaths in all other major forms
    (D)   Does graduate student contact with                      of transportation.
          undergraduates’ work make the grades              (E)   Last year the average number of
          assigned by regular faculty less valid?                 passengers per flown plane was
    (E)   Are regular faculty members in favor of                 significantly lower than that of previous
          giving graduate assistants some teaching                years.
          responsibility?




                                                                        GO ON TO THE NEXT PAGE

                                                                                                          179
P R A C T I C E S E T S A N D E X P L A N AT I O N S



7. According to a recent study, attending a single-      8. Although most people know that exercise is good
   sex high school aids an adolescent’s physical            for the body, few realize the extent to which it is
   growth. Cited as evidence is the finding that            valuable to the mind. The blood circulates more
   during the first two years of high school, the           rapidly after physical exertion, thus allowing all
   average boy in an all-boys school grew five              of the body’s organs to operate more efficiently.
   inches, and the average girl in an all-girls school      This increased activity enables the brain to
   grew four inches.                                        receive more oxygen, thereby creating a higher
                                                            capacity for concentration.
    The answer to which of the following questions
    is needed in order to evaluate the reasoning             The main point in the argument above is that
    presented in the study?
                                                             (A)   the greater the amount of oxygen the brain
    (A)    Why was it that the first two years of high             receives, the better the brain
           school were chosen as the focus of the                  functions.
           study?                                            (B)   exercise is a mental, as well as physical,
    (B)    Did some of the boys in the study grow                  activity.
           less than five inches while they were in          (C)   exercise helps the brain more than it does
           high school?                                            the rest of the body.
    (C)    How much do the average male student              (D)   people can greatly improve their powers of
           and the average female student in a co-                 concentration by exercising more often.
           educational school grow during their first
           two years of high school?                         (E)   exercise serves more than one purpose.
    (D)    Did the girls in the study have as
           nutritious a diet as the boys during the
           time the study was being conducted?
    (E)    What was the average height of the boys
           and the average height of the girls upon
           entering high school?




                                                                         GO ON TO THE NEXT PAGE

 180
CRITICAL REASONING PRACTICE SETS



9. Ernesto:    Sales of VCRs—videocassette            10. Archaeologists have discovered various
               recorders—will decline in the next         paintings on the walls and ceiling of a Chinese
               few years because the saturation           cave whose entrance was blocked by a volcanic
               level among U.S. households has            eruption in the 25th century B.C. and only
               virtually been reached.                    recently cleared by an earthquake. Since the
                                                          paintings depict warriors using Type C bronze
   Milton:     Every year a greater number of             weapons, these archaeologists have concluded
               popular feature films is released on       that Type C bronze weapons were already widely
               cassette—at least 6 per month.             used in this area by 2500 B.C., far earlier than
               Clearly VCR sales will remain              was previously believed.
               constant, if not rise.
                                                          Which of the following pieces of additional
   Which of the following is the best logical             evidence would most seriously weaken the
   evaluation of Milton’s response to Ernesto’s           archaeologists’ conclusion?
   argument?
                                                          (A)   Another entrance to the cave remained
   (A)   He cites evidence that, if true, disproves             clear until a second volcanic eruption
         the evidence cited by Ernesto in drawing               1,000 years after the first.
         his conclusion.
                                                          (B)   Archaeologists have evidence that Type C
   (B)   He points out a gap in the logic followed              bronze weapons were in wide use in areas
         by Ernesto in drawing his conclusion.                  of present-day India as early as 2500 B.C.
   (C)   He cites an issue ignored by Ernesto and         (C)   Alternative methods of dating place the
         which outweighs the issues raised by                   time of the volcanic eruption somewhat
         Ernesto.                                               earlier, at around 3000 B.C.
   (D)   He does not speak to Ernesto’s point             (D)   Most experts believe that Type C bronze
         because he fails to raise the issue of                 weapons were not in use anywhere in
         whether VCR sales may be linked to sales               present-day China until 2000 B.C.
         of other leisure-related products.
                                                          (E)   The paintings were very faded when the
   (E)   He fails to respond to Ernesto’s argument              archaeologists found them, making
         because he assumes that nothing will                   identification of the depicted weapons
         significantly retard the sale of VCR’s,                difficult.
         which was the issue that Ernesto raised.




                                                                      GO ON TO THE NEXT PAGE

                                                                                                        181
P R A C T I C E S E T S A N D E X P L A N AT I O N S



11. Considering the current economy, the                 12. A public health official reported that 60 percent
    introduction of a new brand of cereal is unlikely        of the children at summer school have never had
    to expand total sales of cereal, but rather will         the measles or chicken pox, and that of this 60
    just cause some existing buyers of cereal to             percent not one child has ever been observed to
    switch brands. So it makes no sense for the              eat the cheese served in the school lunches.
    Coolidge Corporation to introduce another brand          From this he concluded that children who
    of cereal, since they will only hurt sales of the        abstain from cheese products will protect
    brands of cereal they already produce.                   themselves from most childhood disease.

    Which of the following, if true, would most              Each of the following, if true, would strengthen
    seriously weaken the argument above?                     the official’s argument EXCEPT:

    (A)    Total sales of cereal will increase as the        (A)   Medically speaking, whatever serves to
           total population increases.                             inhibit measles and chicken pox will
    (B)    Many new brands of cereal sell extremely                generally inhibit the entire spectrum of
           well for the first year of their existence.             childhood diseases.

    (C)    Coolidge Corporation currently produces           (B)   The observations the official carried out
           fewer brands of cereal than do its                      were extremely accurate, and all those
           competitors.                                            observed to abstain from cheese at
                                                                   school did, in fact, abstain.
    (D)    Some cereal buyers regularly switch from
           brand to brand, even when no new brands           (C)   The children’s eating habits are the same
           have been introduced.                                   at school as anywhere else, and those
                                                                   who abstain from cheese products at
    (E)    Research indicates that the new brand will              school do so in general.
           attract more buyers of competitors’
           cereals than buyers of other Coolidge             (D)   Recent research has pointed to a
           brands.                                                 deficiency in cheese products as one of
                                                                   the major causes of measles and chicken
                                                                   pox infections.
                                                             (E)   Most cheeses and cheese products harbor
                                                                   bacteria that are known to be causative
                                                                   agents for many childhood diseases, such
                                                                   as measles and chicken pox.




                                                                         GO ON TO THE NEXT PAGE

 182
CRITICAL REASONING PRACTICE SETS



13. A confidential survey revealed that 75 percent of    14. Director:   Our engineers are considering two
    the employees of Company P are dissatisfied                          different sites—one on the Abaco
    with their jobs. However, an investigation into                      River and one on the Bornos River—
    working conditions at the company showed                             for a hydroelectric plant. Although
    nothing uncommonly bad. Therefore, Company                           we have the technical expertise to
    P’s consulting firm concluded that the                               build roughly the same plant in
    employees’ dissatisfaction must result from an                       either place, producing roughly the
    unusually high incidence of psychological                            same amount of electricity per hour,
    problems on their part.                                              building the plant on the Abaco site
                                                                         will cost over twice as much money.
    Each of the following, if true, casts doubt on the                   With our budget currently in deficit,
    consulting firm’s conclusion EXCEPT:                                 we should build the dam at the
                                                                         Bornos site.
    (A)   In the investigation of working conditions,
          no account was taken of the fact that for          Which of the following, if true, best explains the
          the past year many Company P employees             difference in building costs for the two proposed
          worked on a joint venture with Company O,          dam sites?
          at Company O’s facilities.
    (B)   Workers in many companies are                      (A)   Many farms along the lower Bornos River
          dissatisfied although there are no                       valley would benefit from the controlled
          apparent problems with their working                     flow of water a dam would make possible.
          conditions.                                        (B)   The Abaco site is in an inaccessible area,
    (C)   The consulting firm’s conception of what                 requiring the building of new roads and the
          constitutes uncommonly bad working                       importation of laborers.
          conditions is not identical to that of             (C)   The Bornos site is near a large city whose
          Company P’s employees.                                   residents could use the resulting lake for
    (D)   The reasons given by Company P’s                         inexpensive recreation.
          employees for their dissatisfaction varied         (D)   The Abaco site is in an area that contains
          greatly from employee to employee.                       many endangered species that would be
    (E)   A battery of sets performed on Company                   threatened by the new dam.
          P’s employees one month ago revealed no            (E)   The Abaco River has a relatively low
          significant psychological stresses or                    volume of flow, making it impossible to
          problems.                                                expand an Abaco plant to meet future
                                                                   electricity needs.




                                                                         GO ON TO THE NEXT PAGE

                                                                                                          183
P R A C T I C E S E T S A N D E X P L A N AT I O N S



15. In 1988, 50 people with emotional disturbances       16. The cause of the peculiar columnar growth
    underwent hypnosis to be cured of their mood             pattern displayed by junipers growing near
    swings. A followup survey in 1993 revealed that          burning underground veins of lignite coal has
    five had fairly stable emotional conditions at the       never been convincingly explained. Until recently,
    time of the survey. These five subjects can              the accepted theory posited that the abundance
    therefore serve as models of the types of                of carbon monoxide in the local atmosphere
    people for whom hypnosis is likely to be                 caused the columnar growth. However, a new
    successful.                                              theory holds that the cause is the persistent
                                                             heat present near these underground fires
    Which of the following, if true, casts the most          which, while not intense enough to inflame the
    doubt on the suitability of those five subjects as       trees, can nonetheless change their normal
    models in the sense described?                           growth pattern.
    (A)    The five subjects have very different             The existence of which of the following would
           personalities and backgrounds.                    provide the strongest support for the new
    (B)    Since 1988, the five subjects have                theory?
           experienced dramatic mood swings
           interspersed with periods of relative             (A)   A columnar juniper growing in an
           stability.                                              atmosphere of intense heat and an
                                                                   absence of carbon monoxide
    (C)    Those people who were still suffering from
           unstable emotional conditions at the time         (B)   A normal juniper growing in an atmosphere
           of the 1993 survey had shown no                         of intense heat and an absence of carbon
           improvement since 1988.                                 monoxide

    (D)    Many psychologists are less concerned             (C)   A columnar juniper growing in an
           about a patient’s mood swings than about                atmosphere of normal heat and a high
           the patient’s willingness to express his or             concentration of carbon monoxide
           her problems and fears.                           (D)   A normal juniper growing in an atmosphere
    (E)    The emotional condition of most of the 45               of intense heat and a high concentration
           subjects who were still unstable at the                 of carbon monoxide
           time of the 1993 survey had actually              (E)   A columnar juniper growing in an
           worsened since 1988.                                    atmosphere of intense heat and a high
                                                                   concentration of carbon monoxide




                                                                              STOP! END OF SET

 184
ANSWERS ON
THE NEXT PAGE
P R A C T I C E S E T S A N D E X P L A N AT I O N S



ANSWER KEY
1. C
2. D
3. C
4. A
5. B
6. A
7. C
8. E
9. E
10. A
11. E
12. D
13. D
14. B
15. B
16. A




186
CRITICAL REASONING SET 2
          Time: 25 minutes—16 Questions
          Directions: For each question in this section, select the best of the answer choices given.

1. Although air pollution was previously thought to    2. Statistics show that although consumption of
   exist almost exclusively in our nation’s cities,       low-calorie, alternative sweeteners has gone up
   the recent increase in the number of persons           in each of the past five years, so has the
   suffering from illnesses attributed to excessive       percentage of the population that is obese.
   air pollution leaves us no choice but to conclude      According to sugar manufacturers, this shows
   that other, nonurban areas are now affected.           that the low-calorie, alternative sweeteners are
                                                          not effective weight loss aids.
    Which of the following, if true, would most
    seriously weaken the conclusion of the argument        Which of the following assertions, if true, would
    above?                                                 most weaken the sugar manufacturers’
    (A)   The nation’s cities have seen a marked           conclusion?
          decrease in levels of air pollution.             (A)   Many people who use low-calorie,
    (B)   The nation has experienced a sharp                     alternative sweeteners eat some foods
          decrease in the number of people moving                that are not low-calorie.
          out of its cities.                               (B)   Some low-calorie, alternative sweeteners
    (C)   Illnesses due to air pollution are among               can increase the appetite, making a
          the least common causes of death to                    person eat more than he or she normally
          urban dwellers.                                        would.

    (D)   Many illnesses previously thought                (C)   Many people use low-calorie, alternative
          unrelated to air pollution are now                     sweeteners to accompany a well-
          considered to be caused by it.                         balanced, low-calorie diet.

    (E)   As a result of the problems in urban             (D)   Obesity has declined among people who
          areas, nonurban areas have passed strict               have consistently used low-calorie,
          pollution control measures.                            alternative sweeteners.
                                                           (E)   The rise in the consumption of low-calorie,
                                                                 alternative sweeteners is primarily due to
                                                                 an increase in the number of users rather
                                                                 than an increase in the amount each user
                                                                 consumes.


                                                                       GO ON TO THE NEXT PAGE

                                                                                                        187
P R A C T I C E S E T S A N D E X P L A N AT I O N S



3. It has long been a commonplace in medical          4. The candy manufacturer’s claim that employee
   literature that the ingestion of drug L, in           “theft” costs the company thousands of dollars
   combination with the application of lotion M,         a year in potential sales is greatly overstated.
   causes the appearance of adverse reaction O.          Most of the candy eaten on the job and not paid
   Recently, however, doubts have been cast on the       for is eaten one piece at a time, by workers who
   role of lotion M in the appearance of adverse         would not be willing to buy an entire box of it
   reaction O.                                           anyway.

   Which one of the following research findings           Which of the following, if true, most weakens the
   could most reasonably have created the doubts          argument above?
   referred to above?                                     (A)   The workers eat only defective candies
   (A)    The appearance of adverse reaction O                  that could not be sold.
          following the ingestion of drug L and the       (B)   Candy eaten by employees represents lost
          application of lotion M                               potential sales to nonemployees.
   (B)    The absence of adverse reaction O               (C)   A few workers account for most of the
          following the ingestion of drug L and the             candy that is eaten but not paid for.
          application of lotion M
                                                          (D)   Most of the candies eaten by employees
   (C)    The ingestion of drug L and the                       are consumed during the holiday season,
          appearance of adverse reaction O in the               when production outputs are at their
          absence of lotion M                                   highest.
   (D)    The absence of adverse reaction O               (E)   The amount of candy eaten by employees
          following the ingestion of drug L without             is only a small fraction of the candy sold
          the application of lotion M                           by the company.
   (E)    The disappearance of adverse reaction O
          following the ingestion of drug L and the
          application of lotion M




                                                                      GO ON TO THE NEXT PAGE

 188
CRITICAL REASONING PRACTICE SETS



5. U.S. officials complain that the country’s trade     6. Children who attend private high schools may
   deficit with Japan is due to the fact that Japan’s      initially feel that they can succeed without doing
   markets are not open enough to imports and              the work required, but as they grow older they
   investment. Japanese officials reply that the           realize the necessity of serious study. Each year
   United States should concentrate on improving           the overwhelming majority of students
   its school systems and investing more money in          disciplined for plagiarism and cheating on their
   scientific research and worker training.                exams is found in the freshman class.

    It can be inferred from the statements above            The argument above would be most weakened if
    that the Japanese officials most probably hold          which of the following were true?
    which of the following opinions?                        (A)   As they move up in grade, students learn
    (A)   The United States should open its own                   how to cheat without being caught.
          markets to more imports and investment.           (B)   First-time offenders for plagiarism and
    (B)   The trade deficit between the United                    cheating on exams are not disciplined.
          States and Japan is more the result of            (C)   The proctors for freshman exams are the
          poor American industrial performance than               least vigilant.
          Japan’s import restrictions.
                                                            (D)   Acts of vandalism are most often
    (C)   The trade deficit between the United                    committed by members of the sophomore
          States and Japan is a result of Japan’s                 class.
          closed markets.
                                                            (E)   Public school students are no less likely
    (D)   U.S. school systems foster a mistrust of                than private school students to believe
          Japan that prevents U.S. businesspeople                 that they can succeed in life without
          from negotiating intelligently with Japan.              working hard.
    (E)   Better education and worker training can
          help shrink the trade imbalance, but
          should not be counted on to close the gap
          entirely.




                                                                        GO ON TO THE NEXT PAGE

                                                                                                         189
P R A C T I C E S E T S A N D E X P L A N AT I O N S



7. The conflict between an artist’s work and the          8. To improve the physical fitness of its students,
   context in which it is placed is a traditional            School District 4 instituted a policy whereby
   problem in aesthetics. Recent exhibits have               students would be given extra credit in physical
   given it a new urgency. Too often a painter’s             education for extracurricular athletic activities.
   canvases have been hung in an improper context            School officials call the program a success,
   because the gallery managers have not                     since participation in after school sports has
   understood what the painter envisioned as the             doubled since the program was instituted.
   work’s proper environment.
                                                              Which of the following, if true, most seriously
   As an attempt to solve the problem described               weakens the claim of the school officials?
   above, it would be most reasonable to                      (A)   Most students who joined after school
   (A)    bring artists and gallery managers into                   sports did so only to get extra credit.
          closer contact, so as to increase the               (B)   Most children who are in poor physical
          artist’s input into the way the exhibit is                condition cannot be persuaded to join
          held                                                      after school sports by such an incentive
   (B)    provide brochures at the exhibit that                     program.
          describe the artist and how he or she               (C)   Few students who joined after school
          intended the exhibit to look                              sports during the extra credit program will
   (C)    redesign galleries so that their decor                    continue to play the sport after the school
          contains nothing that would distract the                  year ends.
          audience from the works themselves                  (D)   Most of the new athletes are students
   (D)    provide a uniform environment for all the                 who had never before participated in after
          works in an exhibition so that they appear                school sports.
          within the same context                             (E)   Fitness set show no significant
   (E)    instruct gallery managers in the fine                     improvement in the physical condition of
          points of aesthetic theory so that they will              students after they join after school
          be able to tell what, if anything, a painting             sports.
          means




                                                                          GO ON TO THE NEXT PAGE

190
CRITICAL REASONING PRACTICE SETS



9. It takes 4 weeks for a team of 5 professional        10. Cultural anthropologists who have been
   window washers working regular full-time hours           observing and interviewing customers in retail
   to properly clean every window of the Empire             stores have announced a definitive theory of
   State Building. The building’s owner demands             consumer purchasing behavior.
   that all the windows always be clean. Yet even if
   the 5 washers work consistently throughout their         Which of the following, if true, would be least
   regular work week, they will not be able to finish       likely to represent a benefit of the theory to
   cleaning all the windows before some windows             retailers?
   will again need cleaning.                                (A)   Retailers will be able to eliminate costly
                                                                  product displays that fail to increase
    It can be correctly inferred on the basis of the              sales.
    statements above that which of the following
    must be true?                                           (B)   Retailers will gain insight into how
                                                                  consumers determine whether or not to
    (A)   If an Empire State Building window is to be             buy a particular product.
          kept clean, it must be cleaned by a
          professional window cleaner.                      (C)   The new theory will make consumers
                                                                  aware of how product displays influence
    (B)   The owner’s demand for proper cleaning of               their purchasing decisions.
          all the windows will never be fulfilled.
                                                            (D)   The new theory will determine what types
    (C)   If a team of 5 window washers cleans all                of retail display gimmicks produce a
          the Empire State Building’s windows in                  negative reaction in consumers.
          less than 4 weeks, some of the windows
          will not be properly cleaned.                     (E)   The new theory will explain why consumers
                                                                  often purchase at different stores goods
    (D)   In order to ensure that all of the Empire               that could be bought at just one store.
          State Building’s windows are clean, the
          owner must have his window washers work
          overtime.
    (E)   Some Empire State Building windows must
          be cleaned more frequently than once
          every four weeks if they are to be kept
          clean.




                                                                        GO ON TO THE NEXT PAGE

                                                                                                          191
P R A C T I C E S E T S A N D E X P L A N AT I O N S



11. Archaeologists have recently found, in various        12. Doubling the cost of public transportation to
    grave sites in the Mexican state of Veracruz,             compensate for money lost by declining ridership
    small ceramic animals with attached wheels. At            would be disastrous. The greater expense would
    first, this find might seem to discredit the belief       only further discourage commuters who are
    that the wheel and its uses were unknown in pre-          already dissatisfied with the poor condition of
    Colombian culture. On reflection, however, it             buses and trains. If the fares are increased,
    would seem that the discovery actually bears out          many commuters will choose to drive their cars
    this belief. To be familiar with these toys and yet       instead, causing pollution and traffic congestion.
    not to apply the principle of the wheel to daily          As a result, the city will lose money and become
    tasks such as carting, transportation, and                even more noisy and smog-filled than it is now.
    pottery making must indicate a lack of
    understanding of the wheel and its potential              Which of the following is an assumption made in
    benefits.                                                 advancing the argument above?
                                                              (A)   Commuters who decide to drive instead of
    Which of the following best expresses the                       using public transportation will not share
    argument made in the passage above?                             rides with one another.
    (A)    If the pre-Colombian people of Veracruz            (B)   Commuters will not park their cars in
           had understood the principle of the wheel,               garages and thereby spend more money
           they would not have attached wheels to                   than they would by using buses or trains.
           ceramic animals.
                                                              (C)   The condition of public transportation will
    (B)    If the pre-Colombian people of Veracruz                  not improve as a result of the fare
           had understood the principle of the wheel,               increase.
           they would have adapted it to everyday
           use.                                               (D)   Commuters who use their own cars
                                                                    currently outnumber those who use buses
    (C)    If the pre-Colombian people of Veracruz                  and trains.
           had uses for the wheel in their everyday
           lives, they would have adapted the idea of         (E)   A significant number of people who now
           the wheel from the wheeled ceramic                       use public transportation have cars or can
           figures.                                                 easily obtain them.
    (D)    The pre-Colombian people of Veracruz
           must have known of the wheel and its
           uses because they attached wheels to
           ceramic animals.
    (E)    Since the pre-Colombian people of
           Veracruz did not know of the wheel or its
           uses, the ceramic animals found in the
           grave sites must be the remains of later
           cultures.




                                                                          GO ON TO THE NEXT PAGE

 192
CRITICAL REASONING PRACTICE SETS



13. The cost of transatlantic airfare has nearly         14. Truck driver:     The gasoline tax is too high and
    doubled over the past five years, yet airlines are                         it must be lowered. It has been
    doing a booming business. Clearly, people today                            raised every year for the last
    have more money to spend on vacations than                                 five years, while other sales
    they did five years ago.                                                   taxes have not. If the
                                                                               government persists in unfairly
    All of the following, if true, would weaken the                            penalizing truck drivers, our
    argument above EXCEPT:                                                     increased operating costs will
    (A)   Most people buying transatlantic tickets                             either hurt consumers or put us
          today use them for business trips, so                                out of business.
          airfare is refunded by their companies.
                                                             State official:   But your gasoline tax dollars
    (B)   There are fewer airlines in existence today                          maintain and improve the very
          than five years ago.                                                 roads you depend on. Without
    (C)   People are taking shorter vacations and                              those additional revenues, road
          staying in cheaper hotels than they used                             conditions would deteriorate,
          to.                                                                  costing you and consumers
                                                                               much more in maintenance and
    (D)   Crossing the Atlantic by ship requires
                                                                               repairs.
          more time than most people can afford.
                                                             If the statements made above are true, the best
    (E)   Domestic airline flights have seen a
                                                             characterization of the logical relationship
          steady increase in passengers.
                                                             between the two arguments is that the state
                                                             official’s response
                                                             (A)   points out that the truck driver’s proposal
                                                                   will actually worsen the problem it is
                                                                   intended to solve
                                                             (B)   is circular, assuming the truth of its
                                                                   conclusion in order to justify its conclusion
                                                             (C)   points out that the truck driver is selfish
                                                                   because more people are aided by the
                                                                   gasoline tax than are penalized
                                                             (D)   is merely an attempt to excuse the
                                                                   government’s policies without providing
                                                                   any justification for those policies
                                                             (E)   points to an inherent contradiction
                                                                   between the cause the truck driver cites
                                                                   and the effects the truck driver thinks will
                                                                   follow from the cause




                                                                             GO ON TO THE NEXT PAGE

                                                                                                            193
P R A C T I C E S E T S A N D E X P L A N AT I O N S



15. Archaeologists recently unearthed a prehistoric     16. The study of foreign languages is finally
    statuette, portraying the figure of a woman, that       becoming a serious endeavor in U.S. education.
    had been carved from a mastodon bone. A team            The number of American college students
    of researchers carefully studied the statuette,         enrolled in non-English language courses has
    which they named the Venus of Orleans. Since it         increased by 20 percent over the last five years.
    was similar in shape and design to another bone         Spanish, with over 500,000 students, is the
    carving, the so-called Venus of Grenoble, they          most popular; its enrollment has increased by
    concluded that in all likelihood it was carved at       30 percent. Meanwhile, enrollment in Japanese
    the same time, about 70,000 years ago.                  and Russian has nearly doubled, and now
    Skeptics point out, however, that carbon-14             comprises eight percent of total foreign language
    testing indicates that the recently discovered          study. Clearly there is now an increased interest
    statuette is only about 50,000 years old.               in foreign language study. When these students
                                                            join the work force, they will enhance not only
    Which of the following, if true, would tend most        U.S. businesses’ ability to compete internationally
    to weaken the force of the skeptics’ objection?         but also our country’s reputation abroad.
    (A)    Carbon-14 dating places the age of the
           Venus of Grenoble at 70,000 years.               The answer to which of the following questions
                                                            would be LEAST relevant to evaluating the above
    (B)    No other, similar, statuettes have been          claims?
           found at the site where the Venus of
           Orleans was unearthed.                           (A)   Do students enrolled in foreign language
                                                                  classes continue their studies long enough
    (C)    The carbon-14 dating process is unreliable             to attain competence in those languages?
           for objects dating from before 60,000 BC.
                                                            (B)   By what percentage has overall enrollment
    (D)    The carbon-14 dating process has                       in U.S. colleges and universities increased
           provided unreliable dates for many objects             over the past five years?
           older than 100,000 years.
                                                            (C)   Does a significant number of students of
    (E)    Some speculation persists that the Venus               foreign languages go into professions in
           of Orleans was carved out of the femur or              which the ability to speak other languages
           thigh bone of a prehistoric ox.                        is useful?
                                                            (D)   Has the study of “dead” languages like
                                                                  Latin and Ancient Greek increased at a
                                                                  similar rate to that of modern languages?
                                                            (E)   How does the percentage increase in
                                                                  foreign language enrollment over the past
                                                                  five years compare to previous increases
                                                                  in enrollment?




                                                                              STOP! END OF SET

 194
ANSWERS ON
THE NEXT PAGE
P R A C T I C E S E T S A N D E X P L A N AT I O N S



ANSWER KEY
1. D
2. D
3. C
4. B
5. B
6. A
7. A
8. E
9. E
10. C
11. B
12. E
13. E
14. A
15. C
16. D




196
CRITICAL REASONING SET 3
         Time: 25 minutes—16 Questions
         Directions: For each question in this section, select the best of the answer choices given.

1. In 1980, 13 percent of the Arbican population      2. The Kapoor Meatworks has a virtual monopoly
   moved from urban areas to suburban areas. This        on expensive, gourmet delicacies. In order to
   percentage steadily declined, until, in 1990, it      expand their market, they intend to offer a
   reached 3 percent.                                    budget line of less costly delicacies. Such a
                                                         product is virtually unknown, and they realize
   If the statements above are true, all of the          that its success depends upon a heavy
   following statements concerning Arbicans              advertising campaign. They have decided to
   between 1980 and 1990 could also be true              finance the advertising with the profits from their
   EXCEPT:                                               gourmet line.
   (A)   The number of Arbicans moving from               Which of the following, assuming each is a
         suburban areas to urban areas also               realistic possibility, would pose the most serious
         decreased.                                       obstacle to the Kapoor Meatworks’ project?
   (B)   The Arbican population increased, and so
         did the number of Arbicans moving from           (A)   The introduction of a budget line of
         urban to suburban areas.                               delicacies completely undercuts the sales
                                                                of the gourmet line.
   (C)   The Arbican population decreased, and so
         did the number of Arbicans moving from           (B)   At the start, the company spends more on
         urban to suburban areas.                               advertising than it makes from sales of
                                                                the budget line delicacies.
   (D)   The Arbican population decreased, and the
         number of Arbicans moving from urban to          (C)   When the budget line delicacies grow in
         suburban areas remained the same.                      popularity, competitors enter the budget
                                                                delicacies market and Kapoor does not
   (E)   The Arbican population increased, and the              have a monopoly in that market.
         number of Arbicans moving from urban to
         suburban areas remained the same.                (D)   Many of the consumers who purchase the
                                                                budget line are tempted to try the
                                                                delicacies offered in the gourmet line.
                                                          (E)   Many of the stores that now carry Kapoor’s
                                                                gourmet line of delicacies are exclusive,
                                                                and refuse to carry their budget line.

                                                                      GO ON TO THE NEXT PAGE

                                                                                                       197
P R A C T I C E S E T S A N D E X P L A N AT I O N S



3. Recent statistics make it seem unlikely that the       4. In a nature reserve in India, people are
   total consumption of electricity determines its           sometimes attacked by tigers. It is believed that
   cost to individual consumers. Recent increases            the tigers will only attack people from behind. So
   in total consumption, especially during the hot           for the past few years many workers in the
   summer months, have sometimes been                        reserve have started wearing masks depicting a
   accompanied by a decrease in the cost per unit            human face on the back of their heads. While
   and at other times by an increase.                        many area residents remain skeptical, no worker
                                                             wearing one of these masks has yet been
    Which of the following positions is best                 attacked by a tiger.
    supported by the information presented above?
                                                              Which of the statements below, if true, would
    (A)    It must be the case that the cost of               best support the argument of those who
           electricity to consumers is what                   advocate the use of the mask?
           determines the total consumption.
    (B)    Even though a correlation exists between           (A)   Many workers in the nature reserve who
           the total consumption of electricity and its             do not wear the masks have been
           cost to consumers, no causal relation                    attacked recently by tigers.
           exists.                                            (B)   Workers in other nature reserves who
    (C)    Further investigation into the way these                 wear similar masks have not been
           statistics were gathered is certainly called             attacked recently by tigers.
           for.                                               (C)   No tigers have been spotted on the nature
    (D)    The cost of electricity depends upon                     reserve in recent years.
           something other than the total                     (D)   Many of the workers who wear the masks
           consumption of electricity.                              also sing while they work in order to
    (E)    The cost-per-unit of electricity to                      frighten away any tigers in the area.
           consumers is dependent on the total                (E)   The tigers have often been observed
           electricity consumption.                                 attacking small deer from in front rather
                                                                    than from behind.




                                                                          GO ON TO THE NEXT PAGE

 198
CRITICAL REASONING PRACTICE SETS



5. Since Arlene Hodges was installed as president       6. Although recently introduced with a wave of
   of the Caralis corporation, profits have increased      publicity, combined audio/video receivers are
   by an average of 11 percent per year. During the        proving tough to sell. As a result, the
   tenure of her predecessor, the corporation’s            manufacturers keep advertising, and offering
   profits averaged a yearly increase of only 7            handsome discounts. What the manufacturers
   percent. Obviously Ms. Hodges’ aggressive               have yet to see is that the public’s reluctance
   marketing efforts have caused the acceleration          isn’t due to the price of the product; the public is
   in the growth of Caralis’ profits.                      still debating the even more fundamental
                                                           question of --------.
    Which of the following, if true, would most
    weaken the conclusion drawn above?                      Which of the following best completes the
                                                            passage above?
    (A)   The corporation’s new manufacturing
          plant, constructed in the past year, has          (A)   whether the product is worth its price
          resulted in a 15 percent increase in              (B)   whether there is a use for such a product
          production capacity.
                                                            (C)   whether the models will soon become
    (B)   For each year of Ms. Hodges’ presidency,                obsolete
          the corporation’s financial records show
          an increase in profits over the previous          (D)   whether the uses for such a product
          year.                                                   outweigh its high cost
    (C)   During the tenure of Ms. Hodges’                  (E)   whether more stylish models will appear in
          predecessor, the corporation began an                   the future
          advertising campaign aimed at capturing
          consumers between the ages of 24 and
          35.
    (D)   Since Ms. Hodges became president, the
          corporation has switched the primary
          focus of its advertising from print ads to
          radio and television commercials.
    (E)   Just before he was replaced, Ms. Hodges’
          predecessor directed the acquisition of a
          rival corporation, which has nearly doubled
          the corporation’s yearly revenues.




                                                                        GO ON TO THE NEXT PAGE

                                                                                                           199
P R A C T I C E S E T S A N D E X P L A N AT I O N S



7. Although statistics and definitions are inexact,       Questions 8 and 9 are based on the following.
   educated guesses put the number of refugees
                                                              In spite of what its critics say, the new,
   worldwide at well over 10 million. The
                                                              expensive artificial kidney implant is a boon
   overwhelming majority prefer to return to their
                                                              to modern medicine. These critics should
   native land than to emigrate to a foreign one.
                                                              remember that the first heart transplants
   The millions of refugees from Afghanistan are
                                                              stimulated an enormous amount of beneficial
   sufficient proof: Despite the toll the war and
                                                              medical research, even though the transplant
   subsequent fighting have taken on their country,
                                                              technique was later rejected in favor of less
   very few have applied for permission to
                                                              invasive surgical procedures.
   emigrate.

    Which of the following, if true, would most
    strengthen the argument above?                        8. The author defends the artificial kidney implant
                                                             against the critics by
    (A)    Most refugees are as reluctant to
           emigrate as are the refugees from                  (A)   attacking her opponents’ method of
           Afghanistan.                                             circular reasoning
    (B)    The refugees from Afghanistan fled what            (B)   implying an analogy between the benefits
           they considered political oppression rather              of the artificial kidney implant and the
           than economic disaster.                                  benefits of the early heart transplants
    (C)    Most of the children born to refugees              (C)   pointing out a contradiction implicit in their
           prefer to remain in their adoptive country               criticism of the implant’s high cost
           rather than return to the land their parents       (D)   criticizing the professional objectivity of
           left.                                                    her opponents rather than their claim
    (D)    Although refugees flee their homes for a           (E)   implying an analogy between the critics’
           variety of different reasons, the                        opposition to the artificial kidney implant
           overwhelming majority are looking for                    and their opposition to the early heart
           improved living conditions.                              transplants
    (E)    The number of refugees worldwide has
           risen dramatically over the last ten years.    9. Opponents of the artificial kidney implant could
                                                             best defend their position against the author’s
                                                             argument by pointing out that

                                                              (A)   once perfected, the new artificial kidney
                                                                    implant will cost very little
                                                              (B)   many of those who oppose the new
                                                                    artificial kidney implant have very good
                                                                    reputations in the scientific community
                                                              (C)   the high cost of the new artificial kidney
                                                                    implant does not necessarily mean that its
                                                                    creators are incompetent or profligate
                                                              (D)   the medical research that the artificial
                                                                    kidney implant will stimulate is similar to
                                                                    the research stimulated by the heart
                                                                    transplants
                                                              (E)   the fact that one medical innovation
                                                                    stimulated beneficial research does not
                                                                    mean that all such innovations will do so

                                                                          GO ON TO THE NEXT PAGE

 200
CRITICAL REASONING PRACTICE SETS



10. The population increase over a given year is         11. Chef: An ordinance ought to be passed banning
    always calculated as a percentage of the                       midtown street vendors from selling food
    previous year’s population, with a population                  within a certain proximity to restaurants.
    decrease being recorded as a negative increase.                With their high rents and costs,
    In 1990 Essex County and Union County both                     restaurants cannot be expected to
    experienced a population increase of more than                 compete with the vendors. Even in cases
    3 percent. In 1991 Essex County and Union                      where these vendors sell food completely
    County both experienced a population increase                  different from the restaurant’s bill of fare,
    of 1.5 percent. In 1992 Union County                           the price differential is enough to attract
    experienced a negative increase in population of               to the vendors customers who would
    Ϫ1.7 percent. Essex County had 209,100                         otherwise have eaten in the restaurants.
    residents in 1990 and 209,000 residents in
    1992.                                                    Which of the following, if true, would most
                                                             weaken the argument above?
    If the information above is accurate, which of the
    following must be true?                                  (A)   The food served in midtown restaurants is
                                                                   better than that sold by street vendors.
    (A)   Both counties experienced positive                 (B)   Most street vendors who sell food would
          population increases in the years 1990                   suffer losses from being forced to move to
          and 1991, yet both experienced negative                  other locations.
          population increases in 1992.
                                                             (C)   Most customers who eat in midtown do so
    (B)   In 1992 more people moved from Union                     on expense accounts, and do not pay for
          County to Essex County than moved from                   their own meals.
          Essex County to Union County.
                                                             (D)   There are not enough police officers to
    (C)   In 1990 there were more people living in                 enforce regulations requiring street
          Union County than in Essex County, but in                vendors to move a certain distance from
          1992 there were more people living in                    restaurants.
          Essex County than in Union County.
                                                             (E)   The average profit on food sold by a street
    (D)   Both counties experienced negative                       vendor is roughly the same percentage as
          population increases in the years 1990                   that on food served in restaurants.
          and 1991, yet both experienced positive
          population increases in 1992.
    (E)   Both counties experienced positive
          population increases in the years 1990
          and 1991, yet while Essex County
          experienced a positive increase in 1992,
          Union County had a negative increase.




                                                                          GO ON TO THE NEXT PAGE

                                                                                                           201
P R A C T I C E S E T S A N D E X P L A N AT I O N S



12. It has been against the law for federal agencies      13. The manufacturer of DTF claims its product
    and federal contractors to discriminate against a         reduces facial wrinkles and wishes to sell it as a
    qualified job applicant because of a disability.          pharmaceutical. The Food and Drug
    Now that Congress has approved legislation to             Administration (FDA), however, has ordered
    expand these existing provisions to cover private         lengthy and costly experiments to determine
    industry as well, the number of disabled people           whether DTF truly reduces facial wrinkles. The
    who are involuntarily unemployed will drop                manufacturer, a small cosmetics firm, lacks the
    substantially.                                            resources to carry out the required research and
                                                              will probably comply with less rigorous FDA
    The author of the above argument must be                  regulations by labeling DTF a cosmetic.
    assuming which of the following?
                                                              Which of the following can be most reasonably
    (A)    Many congressmen were reluctant to pass            inferred from the statements above?
           the new legislation to prevent
           discrimination against the disabled.               (A)   Cosmetics are among the products not
    (B)    Some private employers in the past                       regulated by the FDA.
           deliberately chose not to hire qualified but       (B)   Only established pharmaceutical firms
           disabled job applicants.                                 have the capital required to enter new
    (C)    The federal government currently employs                 products in the market.
           more disabled people than does private             (C)   The makers of DTF thought they would sell
           industry.                                                greater quantities of their product if it were
    (D)    The approved legislation would stop                      a pharmaceutical rather than a cosmetic.
           discrimination against the disabled in the         (D)   The FDA regulates claims made about
           public and private sectors.                              pharmaceuticals more strictly than claims
    (E)    Many disabled people voluntarily choose                  made about cosmetics.
           to remain unemployed.                              (E)   DTF has very little effect, if any, on facial
                                                                    wrinkles and would not have received FDA
                                                                    approval.




                                                                               STOP! END OF TEST

 202
CRITICAL REASONING PRACTICE SETS



14. Homeowner:     Recent drops in the value of our     15. Plant Y thrives in environments of great sunlight
                   homes are due to the                     and very little moisture. Desert X is an
                   undesirability of living near the        environment with constant, powerful sunlight,
                   recently opened 24-hour bus              and almost no moisture. Although Plant Y thrives
                   depot.                                   in the areas surrounding Desert X, it does not
                                                            exist naturally in the desert, nor does it survive
   Transportation official: The police, not the bus         long when introduced there.
                   depot, are at fault. Survey data
                   shows that most prospective              Which of the following, if true, would be most
                   homeowners avoid the                     useful in explaining the apparent discrepancy
                   community because of its high            above?
                   crime rate.
                                                            (A)   Desert X’s climate is far too harsh for the
   Which of the following, if true, would be the most             animals that normally feed on Plant Y.
   effective rebuttal that the homeowner could              (B)   For one week in the fall, Desert X gets
   make to the argument put forward by the                        consistent rainfall.
   transportation official?
                                                            (C)   The environment around Desert X is
   (A)   The community’s crime rate has risen                     ideally suited to the needs of Plant Y.
         nearly as quickly as the value of homes            (D)   Due to the lack of sufficient moisture,
         has fallen, over the same period of time.                Desert X can support very little plant life.
   (B)   Those homeowners whose homes are                   (E)   Plant Y cannot survive in temperatures as
         situated in the immediate area of the bus                high as those normally found in Desert X.
         depot must endure constant noise.
   (C)   The constant activity and commerce
         generated by the bus depot has made the
         community a favored hangout for thieves
         and other criminals.
   (D)   The community’s police force has not
         seen a pay raise or increase in manpower
         for the last five years.
   (E)   The surveys of prospective homeowners
         did not include questions concerning the
         desirability of living in a community with a
         24-hour bus depot.




                                                                        GO ON TO THE NEXT PAGE

                                                                                                            203
P R A C T I C E S E T S A N D E X P L A N AT I O N S



16. The World Automobile Association (WAA)
    publishes a list of the “Best and Worst Drivers of
    the World,” ranking the drivers of every nation
    according to the number of traffic deaths per
    mile driven in that country.

    Each of the following, if true, would by itself
    provide a logical objection to using the WAA’s
    ranking as a representation of the quality of
    drivers in each nation EXCEPT:

    (A)    The roads in some countries are in bad
           repair and are therefore more dangerous
           than roads in other countries.
    (B)    The average driver in industrialized
           countries can afford to maintain his or her
           car in better condition than can the
           average driver in less developed countries.
    (C)    Some countries contain hundreds of
           thousands of miles of road while other
           countries contain relatively few miles of
           road.
    (D)    Minor accidents that would cause little
           injury in many countries are often fatal
           when they occur in extremely mountainous
           countries.
    (E)    Because of differences in national
           economies, the average car in some
           countries contains many more passengers
           than does the average car in other
           countries.




                                                                 STOP! END OF SET

 204
ANSWERS ON
THE NEXT PAGE
P R A C T I C E S E T S A N D E X P L A N AT I O N S



ANSWER KEY
1. D
2. A
3. D
4. A
5. E
6. B
7. A
8. B
9. E
10. A
11. C
12. B
13. D
14. C
15. E
16. C




206
CRITICAL
                      REASONING
                    EXPLANATIONS                                           CHAPTER 21




CRITICAL REASONING EXPLANATIONS SET 1

Answer Key:
1. C                    7. C                     13. D
2. D                    8. E                     14. B
3. C                    9. E                     15. B
4. A                    10. A                    16. A
5. B                    11. E
6. A                    12. D


1. C                                                     background. Supervisory experience (B) isn’t the
The Conclusion: Union reform movements should            same as coming up through the ranks. Labor unions
build staffs out of workers who have come up the         having problems, which the author admits, isn’t the
ranks.                                                   same as (D) most of them unfairly representing
                                                         workers’ interests. That’s an overstatement.
The Evidence: Union movements are currently              “Democratic” (E) is a new term, and one the
suffering from a problem: Their staffs consist of        argument doesn’t need.
college-educated professional types who don’t
understand the concerns of the worker.
                                                         2. D
If the author believes that hiring up-from-the-ranks     The Conclusion: None is stated, but as the question
workers (an idea introduced in the conclusion) will      stem alerts us, one is implied.
cure that problem, he must be assuming that these
former workers do understand workers’ real               The Evidence: A plant opened in Country Y during a
concerns.                                                revolution ten years ago has always generated
                                                         substantial profits.
There’s no need to assume that higher education
lessens people’s identification with their class         The author draws an analogy between the two plants.
background (A), since the author hasn’t said that the    since the one in Country Y has made money, so too,
lawyers, economists, and experts who don’t               she implies, will the one in Country X.
understand workers come from a working class             The author’s point is that the plant can be successful



                                                                                                          295
P R A C T I C E S E T S A N D E X P L A N AT I O N S



despite the war, not that (A) the plant will be             The implication is that Zydol is more cost-effective
successful because of the war. The author presents the      than aspirin: you get a much better bargain with a
two countries as similar, so she’s not arguing that (B)     bottle of Zydol. But if the aspirin bottle contains
one is more stable than the other. The author is            more than twice as many tablets as the Zydol bottle,
attacking her opponents’ argument, but not (C) their        then it’s the aspirin bottle that gives you more pain
motives. She makes no judgment (E) as to which              reliever for your money.
plant will be more successful. Remember, comparing
and contrasting things that are considered equivalent       The argument concerns which is the better bargain,
in the stimulus is a common wrong answer type for           not which is the better pain reliever. Thus it doesn’t
inference questions.                                        matter (B) that the ingredient is the same. (C) points
                                                            to some consumers who won’t want Zydol. Perhaps,
                                                            but one, these may be very few, and two, this doesn’t
3. C                                                        attack the author’s reasoning, which is based on cost.
The Conclusion: There is none stated. One is implied,       Pain sufferers who don’t have a doctor’s prescription
though, and we’re to locate it among the answer             are left with the same choice between Zydol and
choices.                                                    aspirin, so (D) doesn’t weaken Zydol’s case against
The Evidence: Since deregulation, low air fares have        aspirin. Even if you make an unwarranted leap and
correlated with increased miles flown. Recently prices      assume that there may be fewer of the large Zydol
have risen, but the number of passenger-miles flown         pills (E) per bottle, choice (A) clearly does a better job
is still increasing.                                        of arguing along this line.

The passage begins with a correlation and then              5. B
breaks it in the next sentence. What can you infer          The Undergraduates’ Conclusion: If grades are
from this? Not very much, you may have thought,             assigned by graduate assistants, regular full-time fac-
and you’d be right: the key to this question is not         ulty will devote less time to undergraduate work.
drawing an unwarranted sweeping conclusion from
the evidence. What you can safely conclude is that the      Their Evidence: There really isn’t any: just the idea
air fare doesn’t, by itself, determine the amount of air    that giving grading responsibilities to graduate
travel.                                                     students will somehow move regular full-time faculty
                                                            away from the undergraduates’ work.
That air-fares don’t have any effect on air travel (A) is
too strong. We can’t infer why passenger-miles flown        The only thing to do is go through the choices to see
have increased (B); it might be at the expense of other     which question we can answer. Since the
forms of transportation, but it might not be. The           undergraduates are against the proposal, it’s safe to
author merely states that the air fares have risen          conclude that they want the full-time faculty to stay
recently, but never mentions a cause (D) like               interested in their work. We have no information
“takeovers.” So far, the recent rise in air fares hasn’t    about the fairness of graduate assistants’ grades (A).
reduced passenger-miles flown, but that doesn’t             All we know about the discussion sections (C) is that
mean (E) that a further increase won’t.                     graduate assistants now hold them; we aren’t told
                                                            whether or not full-time faculty may hold them. (D)
                                                            asks about the validity of the faculty’s grades. We
4. A                                                        know that some people have proposed that graduate
The Conclusion: Consumers can be expected to                assistants be given some grading responsibility, but
switch to Zydol.                                            that doesn’t imply (D) that they believe the faculty’s
                                                            grades have been made less valid by the graduate
The Evidence: Although one has to take two aspirins
                                                            students’ “contact” with undergraduates. And we’re
to get the relief provided by one Zydol, nevertheless a
                                                            told absolutely nothing about (E) the faculty’s
bottle of Zydol costs the same as a bottle of aspirin.
                                                            opinion of the proposal.


 296
C R I T I C A L R E A S O N I N G E X P L A N AT I O N S



6. A                                                       some less. The reasons for the growth difference
The Conclusion: Air travel is becoming more danger-        between boys and girls (D) isn’t relevant to the
ous.                                                       growth differences between coed and single-sex
                                                           schools. Since the study is only interested in how
The Evidence: In the last year there have been seven       much students grow during the first two years of
collisions and over 700 deaths, the highest number of      high-school, it’s irrelevant how tall they are to start
deaths ever.                                               with (E).
We want something that suggests that the increase in
fatalities doesn’t prove an increase in danger. If more    8. E
and more people are traveling by airplane, an increase     The Conclusion: Exercise is good for the mind as well
in the number of deaths doesn’t prove that air travel      as for the body.
is becoming more dangerous. The question of how
dangerous air travel is can’t be answered unless we        The Evidence: Exercise increases the speed at which
know the proportion, not the number, of passengers         blood circulates, allowing the brain to receive more
who get killed.                                            oxygen and thus concentrate better.

The author’s contention isn’t undermined by (B) the        The main point is the conclusion, that exercise is
reasons for the lack of safety. Pointing out more          good for the mind as well as the body. More generally,
dangerous methods of transportation (C) doesn’t            exercise serves more than one purpose.
deny that the danger of air travel is increasing.          (A) goes out on a limb; the author’s point is the
Likewise, the fact that other forms of transportation      beneficial effects of exercise, not the unlimited
are also getting more dangerous (D) doesn’t dent the       benefits of oxygen. While exercise aids mental
author’s claim. Fewer passengers per place (E) means       activity, it’s not (B) a mental activity as such. Whether
more collisions, but we already know how many              exercise aids the brain more or less than he rest of the
collisions there were.                                     body (C) isn’t discussed. This is a classic wrong
                                                           answer type: the choice that compares two items the
7. C                                                       stimulus treats equally. While the author argues that
The Conclusion: Attending a single-sex high school         exercise is beneficial to the mind, “greatly improving”
promotes growth.                                           concentration (D) is an overstatement.

The Evidence: During the first two years, the average
student in an all—boys school grew five inches and         9. E
the average girl in an all—girls school grew four          Ernesto’s Conclusion: VCR sales will decline.
inches.                                                    Ernesto’s Evidence: The saturation level for
The question stem tells you that something has been        households has been reached.
left out of the argument. The conclusion compares          Milton’s Conclusion: VCR sales won’t decline.
single-sex schools to coed schools, but the evidence
only cites data from single-sex schools. Until we          Milton’s Evidence: More and more films are released
know how much the average student grows in co-             on videotape.
educational schools, we have no evidence that the
growth in single-sex schools is greater.                   Milton simply ignores Ernesto’s argument that the
                                                           saturation point has been reached for VCR’s. He
Even if we knew the answer to (A) we’d still have no       concentrates on the new cassettes coming out, but the
evidence from the coed schools. (B) is a rather silly      availability of new cassettes doesn’t mean people
question: we’re dealing with average growth, so we’d       need buy or will buy more recorders. Milton ignores
expect some boys to grow more than the average and         the relevant issue of saturation raised by Ernesto, and



                                                                                                               297
P R A C T I C E S E T S A N D E X P L A N AT I O N S



simply assumes that sales of VCR’s will continue as       11. E
before.                                                   The Conclusion: Introducing a new cereal will only
                                                          hurt the brands Coolidge already produces, so
Milton’s evidence doesn’t disprove Ernesto’s evidence     Coolidge shouldn’t introduce another brand of
(A). The videos appearing each year have nothing to       cereal.
do with whether the saturation point in VCR
ownership has been reached. “Finding a gap” (B) in        The Evidence: The introduction of a new brand of
an argument means demonstrating that the evidence         cereal doesn’t increase the total number of cereal
doesn’t lead to the conclusion; Milton merely brings      buyers, but only encourages those who already buy
up some irrelevant information that has nothing to        cereal to switch brands.
do with Ernesto’s reasoning. Milton does cite an issue
that Ernesto ignored (C), but the claim that this issue   There’s an assumption here: that the new brand will
“outweighs” the issue of saturation is unsupported.       only attract those who currently buy other brands of
Milton does fail to speak to Ernesto’s point (D) but      Coolidge’s cereal. If the new brand steals buyers from
not for the bizarre reason given here. Ernesto has        competitors’ cereals, then it will help Coolidge by
absolutely nothing to say about “other leisure-related    adding to its total sales.
products” there’s no reason for Milton to raise this      Even if the population does increase and total cereal
issue.                                                    sales with it (A), a new brand might still hurt the sales
                                                          of Coolidge’s established cereals. The new cereal may
10. A                                                     sell well (B), but if that just means it’s stealing lots of
The Conclusion: People must already have been using       buyers from other Coolidge cereals, what good is it?
Type C weapons in this area by 2500 B.C.                  The fact that Coolidge has only a few brands (C)
                                                          doesn’t make it likely that a new cereal brand would
The Evidence: Depictions of the weapons exist on the      steal buyers from competitors’ brands rather than
walls of a cave that was sealed off in 2500 B.C.          from other Coolidge brands. Neither does the fact
We need to show that the paintings either don’t           that some brand switching is usual even when a new
depict weapons that were in use or that the paintings     brand isn’t introduced (D).
were made after 2500 B.C. The archaeologists are
assuming that there was only one entrance to the          12. D
cave. If there’s another entrance to the cave that was    The Conclusion: Not eating cheese protects children
only sealed much later, then people could have            from childhood diseases.
entered the cave and made the paintings long after
the first entrance was sealed.                            The Evidence: All the children at summer school who
                                                          have never had measles or chicken pox have also
If anything, the existence of the weapons in India (B)    never eaten the cheese served in the school lunches.
might be considered to strengthen the argument,
since it shows Type C weapons were in existence as        The author deduces a causal relationship: not eating
early as 2500 B.C. Pushing the date of the eruption       cheese leads to protection from childhood diseases. I
back (C) also strengthens the argument by making it       hope you saw that this has numerous holes. Four of
likely that the paintings were done even earlier than     the choices help to fill those holes; the fifth does not.
claimed. Without some evidence to back it up, the         That fifth choice is (D). If research shows that
opinion of the experts (D) isn’t worth much. That         abstaining from cheese products is a major cause of
identification was difficult (E) is not the same as its   some childhood diseases, then the health official’s
being uncertain or controversial.                         claim that children can protect themselves from
                                                          disease by not eating cheese is flat-out wrong.




 298
C R I T I C A L R E A S O N I N G E X P L A N AT I O N S



It’s important that the author be able (A) to infer        Her Evidence: Although the same plant can produce
from measles and chicken pox (in the evidence) to          the same amount of electricity in either site, building
other childhood diseases. Also necessary is that the       at Abaco will cost twice as much money.
observations be accurate (B) and the students’
behavior at home mirror their behavior at school (C).      Looking at the question stem first here really helps
Each of those speaks to the legitimacy of the              you narrow your focus when reading the stimulus.
correlation. Most important, though, is that there         The only thing you’re interested in is why the Abaco
really be (E) a causal connection, rather than just a      site will cost more than twice as much as the Bornos
correlation, between cheese eating and childhood           site. The stimulus doesn’t give you any hint as to how
illness.                                                   the sites are different, so you’ll have to rely on the
                                                           correct answer to provide a complete explanation by
                                                           itself. If the Abaco site is in the middle of nowhere,
13. D                                                      and requires constructing new roads and importing
The Consulting Firm’s Conclusion: The employees’           laborers, then of course it’s going to be more
dissatisfaction is all in their heads.                     expensive to build there.
Its Evidence: 75 percent of the employees are              The benefits of the Bornos plant to farms (A) don’t
dissatisfied. An investigation showed no                   explain its lower cost of construction. Neither do the
uncommonly bad working conditions.                         benefits to nearby city residents (C). Possible
We need the one choice that doesn’t weaken the             environmental damage caused by a plant built at
consulting firm’s conclusion; that’s a good tip that the   Abaco (D) is another reason for building at Bornos,
conclusion isn’t solidly based. The mere fact that the     but it’s a different reason. The problem or issue of
complaints vary doesn’t hurt the firm’s conclusion         expansion (E), like the environment, might merit
that the complaints are based on psychological             consideration, but it’s not a factor in determining the
problems. In fact, it might even strengthen the firm’s     cost of building.
argument: every employee has his or her own mental
hang-up, so every employee comes up with his or her        15. B
own irrational complaint.                                  The Conclusion: These five people can serve as mod-
                                                           els for the type of person who can be helped by hyp-
If many employees had been working at a different          nosis.
company’s facilities (A), then it could be these
facilities, not mental problems, that are responsible      The Evidence: A study showed that these five
for the complaints. If the situation in Company P is       previously disturbed hypnosis subjects had stable
fairly normal (B), there’s no need for the firm to         emotional conditions.
hypothesize an unusual incidence of psychological
problems among the workforce. If the firm’s                The survey only found that at the time of the study
definition of uncommonly bad conditions differs            the five seemed to be doing OK. Remember, these
from that of the workers (C), then the firm may well       people were originally suffering from mood swings;
have overlooked the real causes of employee                maybe the study just caught them on a good day. If
dissatisfaction. And if a battery of tests (E), showed     that’s the case—if since 1988 these people have been
no significant psychological problems, then the very       experiencing dramatic mood swings and occasional
cause cited by the firm is attacked.                       periods of health—then hypnosis hasn’t really helped
                                                           them and they’re not good models.

14. B                                                      The author presented the people as models of
The Director’s Conclusion: We should build the dam         different types of people who can be helped, not as a
at the Bornos site.                                        single model of a single personality type, so they
                                                           needn’t be similar (A). It doesn’t matter that the other



                                                                                                              299
P R A C T I C E S E T S A N D E X P L A N AT I O N S



45 people who underwent hypnosis didn’t get better
(C); the argument is based on and concerns only the
five who were stable. (E) fails to weaken the argument
for the same reason. The concern of many
psychologists (D) is well outside the scope. We need a
statement that speaks about hypnosis and these five
subjects.

16. A
The New Theory’s Conclusion: Heat (from the burn-
ing coal) causes columnar growth in junipers near
burning underground coal veins.

The New Theory’s Evidence: None really, except the
correlation of columnar growth with these areas with
underground fires.

The Old Theory’s Conclusion: The abundance of
carbon monoxide causes columnar growth.

The Old Theory’s Evidence: None really, except the
correlation of columnar growth with these areas with
high carbon monoxide.

When you scan the choices, you see that each presents
a case of the cause with or without the effect or the
effect with or without the cause. Since the two
theories are in opposition, weakening the old theory
is a way of strengthening the new one. Bearing in
mind from the lesson the key issues in a causal
argument, we recognize that a case of columnar
growth where the cause claimed by the new theory
(heat) is present, but the cause claimed by the old
theory (carbon monoxide) is absent strengthens the
new theory at the expense of the old.

A case (B) where we get the new theory’s alleged
cause (intense heat), without the alleged effect
(columnar junipers) is of no help at all. A columnar
juniper in an atmosphere with high carbon
monoxide but no extra heat (C) strengthens the old
theory. A case with both alleged causes without the
expected effect (D) weakens both theories. Likewise
columnar growth in the presence of both causes (E)
does nothing to promote one theory over the other.




 300
CRITICAL REASONING SET 1
CRITICAL REASONING SET 1
CRITICAL REASONING SET 1
CRITICAL REASONING SET 1
CRITICAL REASONING SET 1
CRITICAL REASONING SET 1
CRITICAL REASONING SET 1
CRITICAL REASONING SET 1
CRITICAL REASONING SET 1
CRITICAL REASONING SET 1
CRITICAL REASONING SET 1

More Related Content

Viewers also liked

How to Get a Perfect Score on the GMAT
How to Get a Perfect Score on the GMATHow to Get a Perfect Score on the GMAT
How to Get a Perfect Score on the GMATMagoosh
 
Critical reasoning
Critical reasoningCritical reasoning
Critical reasoningWordpandit
 
GMAT Geometry - everything you need to know
GMAT Geometry - everything you need to know GMAT Geometry - everything you need to know
GMAT Geometry - everything you need to know GMAT Prep Now
 
Critical Reasoning
Critical ReasoningCritical Reasoning
Critical Reasoningguest499798
 
Magoosh's Complete Guide to the GMAT
Magoosh's Complete Guide to the GMATMagoosh's Complete Guide to the GMAT
Magoosh's Complete Guide to the GMATMagoosh
 
Gmat-sentence-correction-guide
Gmat-sentence-correction-guideGmat-sentence-correction-guide
Gmat-sentence-correction-guideRushabh Vora
 
Cr concepts the best resource for gmat cr from ivy-gmat (sandeep gupta)
Cr concepts   the best resource for gmat cr from ivy-gmat (sandeep gupta)Cr concepts   the best resource for gmat cr from ivy-gmat (sandeep gupta)
Cr concepts the best resource for gmat cr from ivy-gmat (sandeep gupta)mamunapece
 
GMAT Critical Reasoning - everything you need to know
GMAT Critical Reasoning - everything you need to know GMAT Critical Reasoning - everything you need to know
GMAT Critical Reasoning - everything you need to know GMAT Prep Now
 
Test driven development in C
Test driven development in CTest driven development in C
Test driven development in CAmritayan Nayak
 
Basics of c++ Programming Language
Basics of c++ Programming LanguageBasics of c++ Programming Language
Basics of c++ Programming LanguageAhmad Idrees
 
The Next Tsunami AI Blockchain IOT and Our Swarm Evolutionary Singularity
The Next Tsunami AI Blockchain IOT and Our Swarm Evolutionary SingularityThe Next Tsunami AI Blockchain IOT and Our Swarm Evolutionary Singularity
The Next Tsunami AI Blockchain IOT and Our Swarm Evolutionary SingularityDinis Guarda
 
Agriculture connectée 4.0
Agriculture connectée 4.0Agriculture connectée 4.0
Agriculture connectée 4.0Jérôme Monteil
 
GMAT Math Flashcards
GMAT Math FlashcardsGMAT Math Flashcards
GMAT Math FlashcardsGMAT Prep Now
 

Viewers also liked (18)

How to Get a Perfect Score on the GMAT
How to Get a Perfect Score on the GMATHow to Get a Perfect Score on the GMAT
How to Get a Perfect Score on the GMAT
 
Critical reasoning
Critical reasoningCritical reasoning
Critical reasoning
 
GMAT Geometry - everything you need to know
GMAT Geometry - everything you need to know GMAT Geometry - everything you need to know
GMAT Geometry - everything you need to know
 
Critical Reasoning
Critical ReasoningCritical Reasoning
Critical Reasoning
 
Manhattan Elite Prep Practical GMAT Test Prep Questions
Manhattan Elite Prep Practical GMAT Test Prep Questions Manhattan Elite Prep Practical GMAT Test Prep Questions
Manhattan Elite Prep Practical GMAT Test Prep Questions
 
Magoosh's Complete Guide to the GMAT
Magoosh's Complete Guide to the GMATMagoosh's Complete Guide to the GMAT
Magoosh's Complete Guide to the GMAT
 
Gmat-sentence-correction-guide
Gmat-sentence-correction-guideGmat-sentence-correction-guide
Gmat-sentence-correction-guide
 
How to Become a Better Speaker
How to Become a Better SpeakerHow to Become a Better Speaker
How to Become a Better Speaker
 
Cr concepts the best resource for gmat cr from ivy-gmat (sandeep gupta)
Cr concepts   the best resource for gmat cr from ivy-gmat (sandeep gupta)Cr concepts   the best resource for gmat cr from ivy-gmat (sandeep gupta)
Cr concepts the best resource for gmat cr from ivy-gmat (sandeep gupta)
 
GMAT Critical Reasoning - everything you need to know
GMAT Critical Reasoning - everything you need to know GMAT Critical Reasoning - everything you need to know
GMAT Critical Reasoning - everything you need to know
 
State of the Cloud 2017
State of the Cloud 2017State of the Cloud 2017
State of the Cloud 2017
 
Test driven development in C
Test driven development in CTest driven development in C
Test driven development in C
 
100 esl-games
100 esl-games100 esl-games
100 esl-games
 
Basics of c++ Programming Language
Basics of c++ Programming LanguageBasics of c++ Programming Language
Basics of c++ Programming Language
 
The Next Tsunami AI Blockchain IOT and Our Swarm Evolutionary Singularity
The Next Tsunami AI Blockchain IOT and Our Swarm Evolutionary SingularityThe Next Tsunami AI Blockchain IOT and Our Swarm Evolutionary Singularity
The Next Tsunami AI Blockchain IOT and Our Swarm Evolutionary Singularity
 
BUSINESS QUIZ -Round 1
 BUSINESS QUIZ -Round 1 BUSINESS QUIZ -Round 1
BUSINESS QUIZ -Round 1
 
Agriculture connectée 4.0
Agriculture connectée 4.0Agriculture connectée 4.0
Agriculture connectée 4.0
 
GMAT Math Flashcards
GMAT Math FlashcardsGMAT Math Flashcards
GMAT Math Flashcards
 

Recently uploaded

Vip Modals Call Girls (Delhi) Rohini 9711199171✔️ Full night Service for one...
Vip  Modals Call Girls (Delhi) Rohini 9711199171✔️ Full night Service for one...Vip  Modals Call Girls (Delhi) Rohini 9711199171✔️ Full night Service for one...
Vip Modals Call Girls (Delhi) Rohini 9711199171✔️ Full night Service for one...shivangimorya083
 
Booking open Available Pune Call Girls Ambegaon Khurd 6297143586 Call Hot In...
Booking open Available Pune Call Girls Ambegaon Khurd  6297143586 Call Hot In...Booking open Available Pune Call Girls Ambegaon Khurd  6297143586 Call Hot In...
Booking open Available Pune Call Girls Ambegaon Khurd 6297143586 Call Hot In...Call Girls in Nagpur High Profile
 
Delhi Call Girls Munirka 9711199171 ☎✔👌✔ Whatsapp Hard And Sexy Vip Call
Delhi Call Girls Munirka 9711199171 ☎✔👌✔ Whatsapp Hard And Sexy Vip CallDelhi Call Girls Munirka 9711199171 ☎✔👌✔ Whatsapp Hard And Sexy Vip Call
Delhi Call Girls Munirka 9711199171 ☎✔👌✔ Whatsapp Hard And Sexy Vip Callshivangimorya083
 
Personal Brand Exploration - Fernando Negron
Personal Brand Exploration - Fernando NegronPersonal Brand Exploration - Fernando Negron
Personal Brand Exploration - Fernando Negronnegronf24
 
Delhi Call Girls Preet Vihar 9711199171 ☎✔👌✔ Whatsapp Body to body massage wi...
Delhi Call Girls Preet Vihar 9711199171 ☎✔👌✔ Whatsapp Body to body massage wi...Delhi Call Girls Preet Vihar 9711199171 ☎✔👌✔ Whatsapp Body to body massage wi...
Delhi Call Girls Preet Vihar 9711199171 ☎✔👌✔ Whatsapp Body to body massage wi...shivangimorya083
 
内布拉斯加大学林肯分校毕业证录取书( 退学 )学位证书硕士
内布拉斯加大学林肯分校毕业证录取书( 退学 )学位证书硕士内布拉斯加大学林肯分校毕业证录取书( 退学 )学位证书硕士
内布拉斯加大学林肯分校毕业证录取书( 退学 )学位证书硕士obuhobo
 
Zeeman Effect normal and Anomalous zeeman effect
Zeeman Effect normal and Anomalous zeeman effectZeeman Effect normal and Anomalous zeeman effect
Zeeman Effect normal and Anomalous zeeman effectPriyanshuRawat56
 
Motilal Oswal Gift City Fund PPT - Apr 2024.pptx
Motilal Oswal Gift City Fund PPT - Apr 2024.pptxMotilal Oswal Gift City Fund PPT - Apr 2024.pptx
Motilal Oswal Gift City Fund PPT - Apr 2024.pptxMaulikVasani1
 
CALL ON ➥8923113531 🔝Call Girls Husainganj Lucknow best Female service 🧳
CALL ON ➥8923113531 🔝Call Girls Husainganj Lucknow best Female service  🧳CALL ON ➥8923113531 🔝Call Girls Husainganj Lucknow best Female service  🧳
CALL ON ➥8923113531 🔝Call Girls Husainganj Lucknow best Female service 🧳anilsa9823
 
Delhi Call Girls South Ex 9711199171 ☎✔👌✔ Whatsapp Hard And Sexy Vip Call
Delhi Call Girls South Ex 9711199171 ☎✔👌✔ Whatsapp Hard And Sexy Vip CallDelhi Call Girls South Ex 9711199171 ☎✔👌✔ Whatsapp Hard And Sexy Vip Call
Delhi Call Girls South Ex 9711199171 ☎✔👌✔ Whatsapp Hard And Sexy Vip Callshivangimorya083
 
TEST BANK For Evidence-Based Practice for Nurses Appraisal and Application of...
TEST BANK For Evidence-Based Practice for Nurses Appraisal and Application of...TEST BANK For Evidence-Based Practice for Nurses Appraisal and Application of...
TEST BANK For Evidence-Based Practice for Nurses Appraisal and Application of...robinsonayot
 
Resumes, Cover Letters, and Applying Online
Resumes, Cover Letters, and Applying OnlineResumes, Cover Letters, and Applying Online
Resumes, Cover Letters, and Applying OnlineBruce Bennett
 
Nandini Layout Call Girls: 🍓 7737669865 🍓 High Profile Model Escorts | Bangal...
Nandini Layout Call Girls: 🍓 7737669865 🍓 High Profile Model Escorts | Bangal...Nandini Layout Call Girls: 🍓 7737669865 🍓 High Profile Model Escorts | Bangal...
Nandini Layout Call Girls: 🍓 7737669865 🍓 High Profile Model Escorts | Bangal...amitlee9823
 
Delhi Call Girls Patparganj 9711199171 ☎✔👌✔ Whatsapp Hard And Sexy Vip Call
Delhi Call Girls Patparganj 9711199171 ☎✔👌✔ Whatsapp Hard And Sexy Vip CallDelhi Call Girls Patparganj 9711199171 ☎✔👌✔ Whatsapp Hard And Sexy Vip Call
Delhi Call Girls Patparganj 9711199171 ☎✔👌✔ Whatsapp Hard And Sexy Vip Callshivangimorya083
 
Virgin Call Girls Delhi Service-oriented sexy call girls ☞ 9899900591 ☜ Rita ...
Virgin Call Girls Delhi Service-oriented sexy call girls ☞ 9899900591 ☜ Rita ...Virgin Call Girls Delhi Service-oriented sexy call girls ☞ 9899900591 ☜ Rita ...
Virgin Call Girls Delhi Service-oriented sexy call girls ☞ 9899900591 ☜ Rita ...poojakaurpk09
 
CALL ON ➥8923113531 🔝Call Girls Gosainganj Lucknow best sexual service
CALL ON ➥8923113531 🔝Call Girls Gosainganj Lucknow best sexual serviceCALL ON ➥8923113531 🔝Call Girls Gosainganj Lucknow best sexual service
CALL ON ➥8923113531 🔝Call Girls Gosainganj Lucknow best sexual serviceanilsa9823
 
Top Rated Pune Call Girls Warje ⟟ 6297143586 ⟟ Call Me For Genuine Sex Servi...
Top Rated  Pune Call Girls Warje ⟟ 6297143586 ⟟ Call Me For Genuine Sex Servi...Top Rated  Pune Call Girls Warje ⟟ 6297143586 ⟟ Call Me For Genuine Sex Servi...
Top Rated Pune Call Girls Warje ⟟ 6297143586 ⟟ Call Me For Genuine Sex Servi...Call Girls in Nagpur High Profile
 
Delhi Call Girls In Atta Market 9711199012 Book Your One night Stand Call Girls
Delhi Call Girls In Atta Market 9711199012 Book Your One night Stand Call GirlsDelhi Call Girls In Atta Market 9711199012 Book Your One night Stand Call Girls
Delhi Call Girls In Atta Market 9711199012 Book Your One night Stand Call Girlsshivangimorya083
 
Escorts Service Cambridge Layout ☎ 7737669865☎ Book Your One night Stand (Ba...
Escorts Service Cambridge Layout  ☎ 7737669865☎ Book Your One night Stand (Ba...Escorts Service Cambridge Layout  ☎ 7737669865☎ Book Your One night Stand (Ba...
Escorts Service Cambridge Layout ☎ 7737669865☎ Book Your One night Stand (Ba...amitlee9823
 

Recently uploaded (20)

Vip Modals Call Girls (Delhi) Rohini 9711199171✔️ Full night Service for one...
Vip  Modals Call Girls (Delhi) Rohini 9711199171✔️ Full night Service for one...Vip  Modals Call Girls (Delhi) Rohini 9711199171✔️ Full night Service for one...
Vip Modals Call Girls (Delhi) Rohini 9711199171✔️ Full night Service for one...
 
Booking open Available Pune Call Girls Ambegaon Khurd 6297143586 Call Hot In...
Booking open Available Pune Call Girls Ambegaon Khurd  6297143586 Call Hot In...Booking open Available Pune Call Girls Ambegaon Khurd  6297143586 Call Hot In...
Booking open Available Pune Call Girls Ambegaon Khurd 6297143586 Call Hot In...
 
Delhi Call Girls Munirka 9711199171 ☎✔👌✔ Whatsapp Hard And Sexy Vip Call
Delhi Call Girls Munirka 9711199171 ☎✔👌✔ Whatsapp Hard And Sexy Vip CallDelhi Call Girls Munirka 9711199171 ☎✔👌✔ Whatsapp Hard And Sexy Vip Call
Delhi Call Girls Munirka 9711199171 ☎✔👌✔ Whatsapp Hard And Sexy Vip Call
 
Personal Brand Exploration - Fernando Negron
Personal Brand Exploration - Fernando NegronPersonal Brand Exploration - Fernando Negron
Personal Brand Exploration - Fernando Negron
 
Delhi Call Girls Preet Vihar 9711199171 ☎✔👌✔ Whatsapp Body to body massage wi...
Delhi Call Girls Preet Vihar 9711199171 ☎✔👌✔ Whatsapp Body to body massage wi...Delhi Call Girls Preet Vihar 9711199171 ☎✔👌✔ Whatsapp Body to body massage wi...
Delhi Call Girls Preet Vihar 9711199171 ☎✔👌✔ Whatsapp Body to body massage wi...
 
内布拉斯加大学林肯分校毕业证录取书( 退学 )学位证书硕士
内布拉斯加大学林肯分校毕业证录取书( 退学 )学位证书硕士内布拉斯加大学林肯分校毕业证录取书( 退学 )学位证书硕士
内布拉斯加大学林肯分校毕业证录取书( 退学 )学位证书硕士
 
Zeeman Effect normal and Anomalous zeeman effect
Zeeman Effect normal and Anomalous zeeman effectZeeman Effect normal and Anomalous zeeman effect
Zeeman Effect normal and Anomalous zeeman effect
 
Motilal Oswal Gift City Fund PPT - Apr 2024.pptx
Motilal Oswal Gift City Fund PPT - Apr 2024.pptxMotilal Oswal Gift City Fund PPT - Apr 2024.pptx
Motilal Oswal Gift City Fund PPT - Apr 2024.pptx
 
CALL ON ➥8923113531 🔝Call Girls Husainganj Lucknow best Female service 🧳
CALL ON ➥8923113531 🔝Call Girls Husainganj Lucknow best Female service  🧳CALL ON ➥8923113531 🔝Call Girls Husainganj Lucknow best Female service  🧳
CALL ON ➥8923113531 🔝Call Girls Husainganj Lucknow best Female service 🧳
 
Delhi Call Girls South Ex 9711199171 ☎✔👌✔ Whatsapp Hard And Sexy Vip Call
Delhi Call Girls South Ex 9711199171 ☎✔👌✔ Whatsapp Hard And Sexy Vip CallDelhi Call Girls South Ex 9711199171 ☎✔👌✔ Whatsapp Hard And Sexy Vip Call
Delhi Call Girls South Ex 9711199171 ☎✔👌✔ Whatsapp Hard And Sexy Vip Call
 
TEST BANK For Evidence-Based Practice for Nurses Appraisal and Application of...
TEST BANK For Evidence-Based Practice for Nurses Appraisal and Application of...TEST BANK For Evidence-Based Practice for Nurses Appraisal and Application of...
TEST BANK For Evidence-Based Practice for Nurses Appraisal and Application of...
 
Resumes, Cover Letters, and Applying Online
Resumes, Cover Letters, and Applying OnlineResumes, Cover Letters, and Applying Online
Resumes, Cover Letters, and Applying Online
 
Nandini Layout Call Girls: 🍓 7737669865 🍓 High Profile Model Escorts | Bangal...
Nandini Layout Call Girls: 🍓 7737669865 🍓 High Profile Model Escorts | Bangal...Nandini Layout Call Girls: 🍓 7737669865 🍓 High Profile Model Escorts | Bangal...
Nandini Layout Call Girls: 🍓 7737669865 🍓 High Profile Model Escorts | Bangal...
 
Delhi Call Girls Patparganj 9711199171 ☎✔👌✔ Whatsapp Hard And Sexy Vip Call
Delhi Call Girls Patparganj 9711199171 ☎✔👌✔ Whatsapp Hard And Sexy Vip CallDelhi Call Girls Patparganj 9711199171 ☎✔👌✔ Whatsapp Hard And Sexy Vip Call
Delhi Call Girls Patparganj 9711199171 ☎✔👌✔ Whatsapp Hard And Sexy Vip Call
 
Sensual Moments: +91 9999965857 Independent Call Girls Paharganj Delhi {{ Mon...
Sensual Moments: +91 9999965857 Independent Call Girls Paharganj Delhi {{ Mon...Sensual Moments: +91 9999965857 Independent Call Girls Paharganj Delhi {{ Mon...
Sensual Moments: +91 9999965857 Independent Call Girls Paharganj Delhi {{ Mon...
 
Virgin Call Girls Delhi Service-oriented sexy call girls ☞ 9899900591 ☜ Rita ...
Virgin Call Girls Delhi Service-oriented sexy call girls ☞ 9899900591 ☜ Rita ...Virgin Call Girls Delhi Service-oriented sexy call girls ☞ 9899900591 ☜ Rita ...
Virgin Call Girls Delhi Service-oriented sexy call girls ☞ 9899900591 ☜ Rita ...
 
CALL ON ➥8923113531 🔝Call Girls Gosainganj Lucknow best sexual service
CALL ON ➥8923113531 🔝Call Girls Gosainganj Lucknow best sexual serviceCALL ON ➥8923113531 🔝Call Girls Gosainganj Lucknow best sexual service
CALL ON ➥8923113531 🔝Call Girls Gosainganj Lucknow best sexual service
 
Top Rated Pune Call Girls Warje ⟟ 6297143586 ⟟ Call Me For Genuine Sex Servi...
Top Rated  Pune Call Girls Warje ⟟ 6297143586 ⟟ Call Me For Genuine Sex Servi...Top Rated  Pune Call Girls Warje ⟟ 6297143586 ⟟ Call Me For Genuine Sex Servi...
Top Rated Pune Call Girls Warje ⟟ 6297143586 ⟟ Call Me For Genuine Sex Servi...
 
Delhi Call Girls In Atta Market 9711199012 Book Your One night Stand Call Girls
Delhi Call Girls In Atta Market 9711199012 Book Your One night Stand Call GirlsDelhi Call Girls In Atta Market 9711199012 Book Your One night Stand Call Girls
Delhi Call Girls In Atta Market 9711199012 Book Your One night Stand Call Girls
 
Escorts Service Cambridge Layout ☎ 7737669865☎ Book Your One night Stand (Ba...
Escorts Service Cambridge Layout  ☎ 7737669865☎ Book Your One night Stand (Ba...Escorts Service Cambridge Layout  ☎ 7737669865☎ Book Your One night Stand (Ba...
Escorts Service Cambridge Layout ☎ 7737669865☎ Book Your One night Stand (Ba...
 

CRITICAL REASONING SET 1

  • 1. CRITICAL REASONING PRACTICE SETS CHAPTER 16 CRITICAL REASONING SET 1 Time: 25 minutes—16 Questions Directions: For each question in this section, select the best of the answer choices given. 1. One problem with labor unions today is that their 2. Opening a plant in war-torn Country X is not top staffs consist of college-trained lawyers, inadvisable, despite what critics of the plan may economists, and labor relations experts who say. Ten years ago we opened our plant in cannot understand the concerns of real workers. Country Y in the middle of a revolution; that One goal of union reform movements should be plant has been generating substantial profits to build staffs out of workers who have come up ever since. from the ranks of the industry involved. Which of the following is the author of the The argument above depends primarily on which argument above most reasonably intending the of the following assumptions? reader to conclude? (A) Higher education lessens people’s (A) Wars are profitable for the author’s identification with their class background. particular business. (B) Union staffs should include more people (B) Country X is a more politically stable with first-hand industrial supervisory nation than is Country Y. experience. (C) Critics of the proposed plant in Country X (C) People who have worked in a given are likely to be biased. industry can understand the concerns of (D) The proposed plant in Country X will workers in that industry. generate profits despite the war. (D) Most labor unions today do not fairly (E) The proposed plant in Country X will be represent workers’ interests. more successful than the plant in Country (E) A goal of union reform movements should Y. be to make unions more democratic. GO ON TO THE NEXT PAGE 177
  • 2. P R A C T I C E S E T S A N D E X P L A N AT I O N S 3. Since the deregulation of the air travel industry 4. One Zydol capsule contains twice the pain in the late 1970’s, air fares have been relatively reliever found in regular aspirin. A consumer will low and the number of passenger miles flown have to take two aspirin in order to get the relief has been increasing. In recent months, however, provided by one Zydol. And since a bottle of air fares have risen but the number of passenger Zydol costs the same as a bottle of regular miles flown is still going up. aspirin, consumers can be expected to switch to Zydol. Which of the following can most reasonably be inferred from the statements above? Which of the following, if true, would most weaken the argument that consumers will be (A) The cost of air travel does not affect the discontinuing the use of regular aspirin and number of passenger miles flown. switching to Zydol? (B) People are now flying to destinations that they formerly reached by train or by other (A) A regular bottle of aspirin contains more modes of transportation. than twice as many capsules as does a bottle of Zydol. (C) Factors other than low air fares must be contributing to the rise in the number of (B) The pain reliever in Zydol is essentially the passenger miles flown. same pain reliever found in regular aspirin. (D) Takeovers in the deregulated air travel (C) Some headache sufferers experience a industry have been responsible for the brief period of dizziness shortly after rise in air fares. taking Zydol but not after taking regular aspirin. (E) Air fares can climb even higher without causing a drop in the number of (D) Neither regular aspirin nor Zydol is as passenger miles flown. effective in the relief of serious pain as are drugs available only by prescription. (E) A Zydol capsule is twice as large as the average aspirin. GO ON TO THE NEXT PAGE 178
  • 3. CRITICAL REASONING PRACTICE SETS 5. At a certain college, graduate teaching 6. Air travel is becoming increasingly more assistants conduct discussion sections but have dangerous. In the last year there have been no input into grading. It has been suggested that seven major collisions resulting in over 700 graduate assistants be given some grading deaths, more deaths than in any previous year. responsibility, but many undergraduates oppose that proposal. They argue that if grades are Which statement, if true, would most weaken assigned by graduate assistants, regular full- the argument above? time faculty will devote less time and attention (A) Since the volume of air traffic has been to undergraduate work. increasing all the time, an increase in the The information in the passage above answers number of deaths due to collisions does which one of the following questions? not necessarily mean greater danger. (B) The increase in collisions can be explained (A) Are grades assigned by graduate teaching by statistical coincidence, hijackings, and assistants inherently as fair as those unusual weather. given by regular faculty? (C) Mortality per passenger mile is lower for (B) Are some undergraduates in favor of air travel than for any kind of surface maintaining the full-time faculty’s interest transportation. in their schoolwork? (D) The increase in deaths due to collision in (C) May regular full-time faculty conduct air travel has proceeded at a rate identical discussion sections at the college? to that for deaths in all other major forms (D) Does graduate student contact with of transportation. undergraduates’ work make the grades (E) Last year the average number of assigned by regular faculty less valid? passengers per flown plane was (E) Are regular faculty members in favor of significantly lower than that of previous giving graduate assistants some teaching years. responsibility? GO ON TO THE NEXT PAGE 179
  • 4. P R A C T I C E S E T S A N D E X P L A N AT I O N S 7. According to a recent study, attending a single- 8. Although most people know that exercise is good sex high school aids an adolescent’s physical for the body, few realize the extent to which it is growth. Cited as evidence is the finding that valuable to the mind. The blood circulates more during the first two years of high school, the rapidly after physical exertion, thus allowing all average boy in an all-boys school grew five of the body’s organs to operate more efficiently. inches, and the average girl in an all-girls school This increased activity enables the brain to grew four inches. receive more oxygen, thereby creating a higher capacity for concentration. The answer to which of the following questions is needed in order to evaluate the reasoning The main point in the argument above is that presented in the study? (A) the greater the amount of oxygen the brain (A) Why was it that the first two years of high receives, the better the brain school were chosen as the focus of the functions. study? (B) exercise is a mental, as well as physical, (B) Did some of the boys in the study grow activity. less than five inches while they were in (C) exercise helps the brain more than it does high school? the rest of the body. (C) How much do the average male student (D) people can greatly improve their powers of and the average female student in a co- concentration by exercising more often. educational school grow during their first two years of high school? (E) exercise serves more than one purpose. (D) Did the girls in the study have as nutritious a diet as the boys during the time the study was being conducted? (E) What was the average height of the boys and the average height of the girls upon entering high school? GO ON TO THE NEXT PAGE 180
  • 5. CRITICAL REASONING PRACTICE SETS 9. Ernesto: Sales of VCRs—videocassette 10. Archaeologists have discovered various recorders—will decline in the next paintings on the walls and ceiling of a Chinese few years because the saturation cave whose entrance was blocked by a volcanic level among U.S. households has eruption in the 25th century B.C. and only virtually been reached. recently cleared by an earthquake. Since the paintings depict warriors using Type C bronze Milton: Every year a greater number of weapons, these archaeologists have concluded popular feature films is released on that Type C bronze weapons were already widely cassette—at least 6 per month. used in this area by 2500 B.C., far earlier than Clearly VCR sales will remain was previously believed. constant, if not rise. Which of the following pieces of additional Which of the following is the best logical evidence would most seriously weaken the evaluation of Milton’s response to Ernesto’s archaeologists’ conclusion? argument? (A) Another entrance to the cave remained (A) He cites evidence that, if true, disproves clear until a second volcanic eruption the evidence cited by Ernesto in drawing 1,000 years after the first. his conclusion. (B) Archaeologists have evidence that Type C (B) He points out a gap in the logic followed bronze weapons were in wide use in areas by Ernesto in drawing his conclusion. of present-day India as early as 2500 B.C. (C) He cites an issue ignored by Ernesto and (C) Alternative methods of dating place the which outweighs the issues raised by time of the volcanic eruption somewhat Ernesto. earlier, at around 3000 B.C. (D) He does not speak to Ernesto’s point (D) Most experts believe that Type C bronze because he fails to raise the issue of weapons were not in use anywhere in whether VCR sales may be linked to sales present-day China until 2000 B.C. of other leisure-related products. (E) The paintings were very faded when the (E) He fails to respond to Ernesto’s argument archaeologists found them, making because he assumes that nothing will identification of the depicted weapons significantly retard the sale of VCR’s, difficult. which was the issue that Ernesto raised. GO ON TO THE NEXT PAGE 181
  • 6. P R A C T I C E S E T S A N D E X P L A N AT I O N S 11. Considering the current economy, the 12. A public health official reported that 60 percent introduction of a new brand of cereal is unlikely of the children at summer school have never had to expand total sales of cereal, but rather will the measles or chicken pox, and that of this 60 just cause some existing buyers of cereal to percent not one child has ever been observed to switch brands. So it makes no sense for the eat the cheese served in the school lunches. Coolidge Corporation to introduce another brand From this he concluded that children who of cereal, since they will only hurt sales of the abstain from cheese products will protect brands of cereal they already produce. themselves from most childhood disease. Which of the following, if true, would most Each of the following, if true, would strengthen seriously weaken the argument above? the official’s argument EXCEPT: (A) Total sales of cereal will increase as the (A) Medically speaking, whatever serves to total population increases. inhibit measles and chicken pox will (B) Many new brands of cereal sell extremely generally inhibit the entire spectrum of well for the first year of their existence. childhood diseases. (C) Coolidge Corporation currently produces (B) The observations the official carried out fewer brands of cereal than do its were extremely accurate, and all those competitors. observed to abstain from cheese at school did, in fact, abstain. (D) Some cereal buyers regularly switch from brand to brand, even when no new brands (C) The children’s eating habits are the same have been introduced. at school as anywhere else, and those who abstain from cheese products at (E) Research indicates that the new brand will school do so in general. attract more buyers of competitors’ cereals than buyers of other Coolidge (D) Recent research has pointed to a brands. deficiency in cheese products as one of the major causes of measles and chicken pox infections. (E) Most cheeses and cheese products harbor bacteria that are known to be causative agents for many childhood diseases, such as measles and chicken pox. GO ON TO THE NEXT PAGE 182
  • 7. CRITICAL REASONING PRACTICE SETS 13. A confidential survey revealed that 75 percent of 14. Director: Our engineers are considering two the employees of Company P are dissatisfied different sites—one on the Abaco with their jobs. However, an investigation into River and one on the Bornos River— working conditions at the company showed for a hydroelectric plant. Although nothing uncommonly bad. Therefore, Company we have the technical expertise to P’s consulting firm concluded that the build roughly the same plant in employees’ dissatisfaction must result from an either place, producing roughly the unusually high incidence of psychological same amount of electricity per hour, problems on their part. building the plant on the Abaco site will cost over twice as much money. Each of the following, if true, casts doubt on the With our budget currently in deficit, consulting firm’s conclusion EXCEPT: we should build the dam at the Bornos site. (A) In the investigation of working conditions, no account was taken of the fact that for Which of the following, if true, best explains the the past year many Company P employees difference in building costs for the two proposed worked on a joint venture with Company O, dam sites? at Company O’s facilities. (B) Workers in many companies are (A) Many farms along the lower Bornos River dissatisfied although there are no valley would benefit from the controlled apparent problems with their working flow of water a dam would make possible. conditions. (B) The Abaco site is in an inaccessible area, (C) The consulting firm’s conception of what requiring the building of new roads and the constitutes uncommonly bad working importation of laborers. conditions is not identical to that of (C) The Bornos site is near a large city whose Company P’s employees. residents could use the resulting lake for (D) The reasons given by Company P’s inexpensive recreation. employees for their dissatisfaction varied (D) The Abaco site is in an area that contains greatly from employee to employee. many endangered species that would be (E) A battery of sets performed on Company threatened by the new dam. P’s employees one month ago revealed no (E) The Abaco River has a relatively low significant psychological stresses or volume of flow, making it impossible to problems. expand an Abaco plant to meet future electricity needs. GO ON TO THE NEXT PAGE 183
  • 8. P R A C T I C E S E T S A N D E X P L A N AT I O N S 15. In 1988, 50 people with emotional disturbances 16. The cause of the peculiar columnar growth underwent hypnosis to be cured of their mood pattern displayed by junipers growing near swings. A followup survey in 1993 revealed that burning underground veins of lignite coal has five had fairly stable emotional conditions at the never been convincingly explained. Until recently, time of the survey. These five subjects can the accepted theory posited that the abundance therefore serve as models of the types of of carbon monoxide in the local atmosphere people for whom hypnosis is likely to be caused the columnar growth. However, a new successful. theory holds that the cause is the persistent heat present near these underground fires Which of the following, if true, casts the most which, while not intense enough to inflame the doubt on the suitability of those five subjects as trees, can nonetheless change their normal models in the sense described? growth pattern. (A) The five subjects have very different The existence of which of the following would personalities and backgrounds. provide the strongest support for the new (B) Since 1988, the five subjects have theory? experienced dramatic mood swings interspersed with periods of relative (A) A columnar juniper growing in an stability. atmosphere of intense heat and an absence of carbon monoxide (C) Those people who were still suffering from unstable emotional conditions at the time (B) A normal juniper growing in an atmosphere of the 1993 survey had shown no of intense heat and an absence of carbon improvement since 1988. monoxide (D) Many psychologists are less concerned (C) A columnar juniper growing in an about a patient’s mood swings than about atmosphere of normal heat and a high the patient’s willingness to express his or concentration of carbon monoxide her problems and fears. (D) A normal juniper growing in an atmosphere (E) The emotional condition of most of the 45 of intense heat and a high concentration subjects who were still unstable at the of carbon monoxide time of the 1993 survey had actually (E) A columnar juniper growing in an worsened since 1988. atmosphere of intense heat and a high concentration of carbon monoxide STOP! END OF SET 184
  • 10. P R A C T I C E S E T S A N D E X P L A N AT I O N S ANSWER KEY 1. C 2. D 3. C 4. A 5. B 6. A 7. C 8. E 9. E 10. A 11. E 12. D 13. D 14. B 15. B 16. A 186
  • 11. CRITICAL REASONING SET 2 Time: 25 minutes—16 Questions Directions: For each question in this section, select the best of the answer choices given. 1. Although air pollution was previously thought to 2. Statistics show that although consumption of exist almost exclusively in our nation’s cities, low-calorie, alternative sweeteners has gone up the recent increase in the number of persons in each of the past five years, so has the suffering from illnesses attributed to excessive percentage of the population that is obese. air pollution leaves us no choice but to conclude According to sugar manufacturers, this shows that other, nonurban areas are now affected. that the low-calorie, alternative sweeteners are not effective weight loss aids. Which of the following, if true, would most seriously weaken the conclusion of the argument Which of the following assertions, if true, would above? most weaken the sugar manufacturers’ (A) The nation’s cities have seen a marked conclusion? decrease in levels of air pollution. (A) Many people who use low-calorie, (B) The nation has experienced a sharp alternative sweeteners eat some foods decrease in the number of people moving that are not low-calorie. out of its cities. (B) Some low-calorie, alternative sweeteners (C) Illnesses due to air pollution are among can increase the appetite, making a the least common causes of death to person eat more than he or she normally urban dwellers. would. (D) Many illnesses previously thought (C) Many people use low-calorie, alternative unrelated to air pollution are now sweeteners to accompany a well- considered to be caused by it. balanced, low-calorie diet. (E) As a result of the problems in urban (D) Obesity has declined among people who areas, nonurban areas have passed strict have consistently used low-calorie, pollution control measures. alternative sweeteners. (E) The rise in the consumption of low-calorie, alternative sweeteners is primarily due to an increase in the number of users rather than an increase in the amount each user consumes. GO ON TO THE NEXT PAGE 187
  • 12. P R A C T I C E S E T S A N D E X P L A N AT I O N S 3. It has long been a commonplace in medical 4. The candy manufacturer’s claim that employee literature that the ingestion of drug L, in “theft” costs the company thousands of dollars combination with the application of lotion M, a year in potential sales is greatly overstated. causes the appearance of adverse reaction O. Most of the candy eaten on the job and not paid Recently, however, doubts have been cast on the for is eaten one piece at a time, by workers who role of lotion M in the appearance of adverse would not be willing to buy an entire box of it reaction O. anyway. Which one of the following research findings Which of the following, if true, most weakens the could most reasonably have created the doubts argument above? referred to above? (A) The workers eat only defective candies (A) The appearance of adverse reaction O that could not be sold. following the ingestion of drug L and the (B) Candy eaten by employees represents lost application of lotion M potential sales to nonemployees. (B) The absence of adverse reaction O (C) A few workers account for most of the following the ingestion of drug L and the candy that is eaten but not paid for. application of lotion M (D) Most of the candies eaten by employees (C) The ingestion of drug L and the are consumed during the holiday season, appearance of adverse reaction O in the when production outputs are at their absence of lotion M highest. (D) The absence of adverse reaction O (E) The amount of candy eaten by employees following the ingestion of drug L without is only a small fraction of the candy sold the application of lotion M by the company. (E) The disappearance of adverse reaction O following the ingestion of drug L and the application of lotion M GO ON TO THE NEXT PAGE 188
  • 13. CRITICAL REASONING PRACTICE SETS 5. U.S. officials complain that the country’s trade 6. Children who attend private high schools may deficit with Japan is due to the fact that Japan’s initially feel that they can succeed without doing markets are not open enough to imports and the work required, but as they grow older they investment. Japanese officials reply that the realize the necessity of serious study. Each year United States should concentrate on improving the overwhelming majority of students its school systems and investing more money in disciplined for plagiarism and cheating on their scientific research and worker training. exams is found in the freshman class. It can be inferred from the statements above The argument above would be most weakened if that the Japanese officials most probably hold which of the following were true? which of the following opinions? (A) As they move up in grade, students learn (A) The United States should open its own how to cheat without being caught. markets to more imports and investment. (B) First-time offenders for plagiarism and (B) The trade deficit between the United cheating on exams are not disciplined. States and Japan is more the result of (C) The proctors for freshman exams are the poor American industrial performance than least vigilant. Japan’s import restrictions. (D) Acts of vandalism are most often (C) The trade deficit between the United committed by members of the sophomore States and Japan is a result of Japan’s class. closed markets. (E) Public school students are no less likely (D) U.S. school systems foster a mistrust of than private school students to believe Japan that prevents U.S. businesspeople that they can succeed in life without from negotiating intelligently with Japan. working hard. (E) Better education and worker training can help shrink the trade imbalance, but should not be counted on to close the gap entirely. GO ON TO THE NEXT PAGE 189
  • 14. P R A C T I C E S E T S A N D E X P L A N AT I O N S 7. The conflict between an artist’s work and the 8. To improve the physical fitness of its students, context in which it is placed is a traditional School District 4 instituted a policy whereby problem in aesthetics. Recent exhibits have students would be given extra credit in physical given it a new urgency. Too often a painter’s education for extracurricular athletic activities. canvases have been hung in an improper context School officials call the program a success, because the gallery managers have not since participation in after school sports has understood what the painter envisioned as the doubled since the program was instituted. work’s proper environment. Which of the following, if true, most seriously As an attempt to solve the problem described weakens the claim of the school officials? above, it would be most reasonable to (A) Most students who joined after school (A) bring artists and gallery managers into sports did so only to get extra credit. closer contact, so as to increase the (B) Most children who are in poor physical artist’s input into the way the exhibit is condition cannot be persuaded to join held after school sports by such an incentive (B) provide brochures at the exhibit that program. describe the artist and how he or she (C) Few students who joined after school intended the exhibit to look sports during the extra credit program will (C) redesign galleries so that their decor continue to play the sport after the school contains nothing that would distract the year ends. audience from the works themselves (D) Most of the new athletes are students (D) provide a uniform environment for all the who had never before participated in after works in an exhibition so that they appear school sports. within the same context (E) Fitness set show no significant (E) instruct gallery managers in the fine improvement in the physical condition of points of aesthetic theory so that they will students after they join after school be able to tell what, if anything, a painting sports. means GO ON TO THE NEXT PAGE 190
  • 15. CRITICAL REASONING PRACTICE SETS 9. It takes 4 weeks for a team of 5 professional 10. Cultural anthropologists who have been window washers working regular full-time hours observing and interviewing customers in retail to properly clean every window of the Empire stores have announced a definitive theory of State Building. The building’s owner demands consumer purchasing behavior. that all the windows always be clean. Yet even if the 5 washers work consistently throughout their Which of the following, if true, would be least regular work week, they will not be able to finish likely to represent a benefit of the theory to cleaning all the windows before some windows retailers? will again need cleaning. (A) Retailers will be able to eliminate costly product displays that fail to increase It can be correctly inferred on the basis of the sales. statements above that which of the following must be true? (B) Retailers will gain insight into how consumers determine whether or not to (A) If an Empire State Building window is to be buy a particular product. kept clean, it must be cleaned by a professional window cleaner. (C) The new theory will make consumers aware of how product displays influence (B) The owner’s demand for proper cleaning of their purchasing decisions. all the windows will never be fulfilled. (D) The new theory will determine what types (C) If a team of 5 window washers cleans all of retail display gimmicks produce a the Empire State Building’s windows in negative reaction in consumers. less than 4 weeks, some of the windows will not be properly cleaned. (E) The new theory will explain why consumers often purchase at different stores goods (D) In order to ensure that all of the Empire that could be bought at just one store. State Building’s windows are clean, the owner must have his window washers work overtime. (E) Some Empire State Building windows must be cleaned more frequently than once every four weeks if they are to be kept clean. GO ON TO THE NEXT PAGE 191
  • 16. P R A C T I C E S E T S A N D E X P L A N AT I O N S 11. Archaeologists have recently found, in various 12. Doubling the cost of public transportation to grave sites in the Mexican state of Veracruz, compensate for money lost by declining ridership small ceramic animals with attached wheels. At would be disastrous. The greater expense would first, this find might seem to discredit the belief only further discourage commuters who are that the wheel and its uses were unknown in pre- already dissatisfied with the poor condition of Colombian culture. On reflection, however, it buses and trains. If the fares are increased, would seem that the discovery actually bears out many commuters will choose to drive their cars this belief. To be familiar with these toys and yet instead, causing pollution and traffic congestion. not to apply the principle of the wheel to daily As a result, the city will lose money and become tasks such as carting, transportation, and even more noisy and smog-filled than it is now. pottery making must indicate a lack of understanding of the wheel and its potential Which of the following is an assumption made in benefits. advancing the argument above? (A) Commuters who decide to drive instead of Which of the following best expresses the using public transportation will not share argument made in the passage above? rides with one another. (A) If the pre-Colombian people of Veracruz (B) Commuters will not park their cars in had understood the principle of the wheel, garages and thereby spend more money they would not have attached wheels to than they would by using buses or trains. ceramic animals. (C) The condition of public transportation will (B) If the pre-Colombian people of Veracruz not improve as a result of the fare had understood the principle of the wheel, increase. they would have adapted it to everyday use. (D) Commuters who use their own cars currently outnumber those who use buses (C) If the pre-Colombian people of Veracruz and trains. had uses for the wheel in their everyday lives, they would have adapted the idea of (E) A significant number of people who now the wheel from the wheeled ceramic use public transportation have cars or can figures. easily obtain them. (D) The pre-Colombian people of Veracruz must have known of the wheel and its uses because they attached wheels to ceramic animals. (E) Since the pre-Colombian people of Veracruz did not know of the wheel or its uses, the ceramic animals found in the grave sites must be the remains of later cultures. GO ON TO THE NEXT PAGE 192
  • 17. CRITICAL REASONING PRACTICE SETS 13. The cost of transatlantic airfare has nearly 14. Truck driver: The gasoline tax is too high and doubled over the past five years, yet airlines are it must be lowered. It has been doing a booming business. Clearly, people today raised every year for the last have more money to spend on vacations than five years, while other sales they did five years ago. taxes have not. If the government persists in unfairly All of the following, if true, would weaken the penalizing truck drivers, our argument above EXCEPT: increased operating costs will (A) Most people buying transatlantic tickets either hurt consumers or put us today use them for business trips, so out of business. airfare is refunded by their companies. State official: But your gasoline tax dollars (B) There are fewer airlines in existence today maintain and improve the very than five years ago. roads you depend on. Without (C) People are taking shorter vacations and those additional revenues, road staying in cheaper hotels than they used conditions would deteriorate, to. costing you and consumers much more in maintenance and (D) Crossing the Atlantic by ship requires repairs. more time than most people can afford. If the statements made above are true, the best (E) Domestic airline flights have seen a characterization of the logical relationship steady increase in passengers. between the two arguments is that the state official’s response (A) points out that the truck driver’s proposal will actually worsen the problem it is intended to solve (B) is circular, assuming the truth of its conclusion in order to justify its conclusion (C) points out that the truck driver is selfish because more people are aided by the gasoline tax than are penalized (D) is merely an attempt to excuse the government’s policies without providing any justification for those policies (E) points to an inherent contradiction between the cause the truck driver cites and the effects the truck driver thinks will follow from the cause GO ON TO THE NEXT PAGE 193
  • 18. P R A C T I C E S E T S A N D E X P L A N AT I O N S 15. Archaeologists recently unearthed a prehistoric 16. The study of foreign languages is finally statuette, portraying the figure of a woman, that becoming a serious endeavor in U.S. education. had been carved from a mastodon bone. A team The number of American college students of researchers carefully studied the statuette, enrolled in non-English language courses has which they named the Venus of Orleans. Since it increased by 20 percent over the last five years. was similar in shape and design to another bone Spanish, with over 500,000 students, is the carving, the so-called Venus of Grenoble, they most popular; its enrollment has increased by concluded that in all likelihood it was carved at 30 percent. Meanwhile, enrollment in Japanese the same time, about 70,000 years ago. and Russian has nearly doubled, and now Skeptics point out, however, that carbon-14 comprises eight percent of total foreign language testing indicates that the recently discovered study. Clearly there is now an increased interest statuette is only about 50,000 years old. in foreign language study. When these students join the work force, they will enhance not only Which of the following, if true, would tend most U.S. businesses’ ability to compete internationally to weaken the force of the skeptics’ objection? but also our country’s reputation abroad. (A) Carbon-14 dating places the age of the Venus of Grenoble at 70,000 years. The answer to which of the following questions would be LEAST relevant to evaluating the above (B) No other, similar, statuettes have been claims? found at the site where the Venus of Orleans was unearthed. (A) Do students enrolled in foreign language classes continue their studies long enough (C) The carbon-14 dating process is unreliable to attain competence in those languages? for objects dating from before 60,000 BC. (B) By what percentage has overall enrollment (D) The carbon-14 dating process has in U.S. colleges and universities increased provided unreliable dates for many objects over the past five years? older than 100,000 years. (C) Does a significant number of students of (E) Some speculation persists that the Venus foreign languages go into professions in of Orleans was carved out of the femur or which the ability to speak other languages thigh bone of a prehistoric ox. is useful? (D) Has the study of “dead” languages like Latin and Ancient Greek increased at a similar rate to that of modern languages? (E) How does the percentage increase in foreign language enrollment over the past five years compare to previous increases in enrollment? STOP! END OF SET 194
  • 20. P R A C T I C E S E T S A N D E X P L A N AT I O N S ANSWER KEY 1. D 2. D 3. C 4. B 5. B 6. A 7. A 8. E 9. E 10. C 11. B 12. E 13. E 14. A 15. C 16. D 196
  • 21. CRITICAL REASONING SET 3 Time: 25 minutes—16 Questions Directions: For each question in this section, select the best of the answer choices given. 1. In 1980, 13 percent of the Arbican population 2. The Kapoor Meatworks has a virtual monopoly moved from urban areas to suburban areas. This on expensive, gourmet delicacies. In order to percentage steadily declined, until, in 1990, it expand their market, they intend to offer a reached 3 percent. budget line of less costly delicacies. Such a product is virtually unknown, and they realize If the statements above are true, all of the that its success depends upon a heavy following statements concerning Arbicans advertising campaign. They have decided to between 1980 and 1990 could also be true finance the advertising with the profits from their EXCEPT: gourmet line. (A) The number of Arbicans moving from Which of the following, assuming each is a suburban areas to urban areas also realistic possibility, would pose the most serious decreased. obstacle to the Kapoor Meatworks’ project? (B) The Arbican population increased, and so did the number of Arbicans moving from (A) The introduction of a budget line of urban to suburban areas. delicacies completely undercuts the sales of the gourmet line. (C) The Arbican population decreased, and so did the number of Arbicans moving from (B) At the start, the company spends more on urban to suburban areas. advertising than it makes from sales of the budget line delicacies. (D) The Arbican population decreased, and the number of Arbicans moving from urban to (C) When the budget line delicacies grow in suburban areas remained the same. popularity, competitors enter the budget delicacies market and Kapoor does not (E) The Arbican population increased, and the have a monopoly in that market. number of Arbicans moving from urban to suburban areas remained the same. (D) Many of the consumers who purchase the budget line are tempted to try the delicacies offered in the gourmet line. (E) Many of the stores that now carry Kapoor’s gourmet line of delicacies are exclusive, and refuse to carry their budget line. GO ON TO THE NEXT PAGE 197
  • 22. P R A C T I C E S E T S A N D E X P L A N AT I O N S 3. Recent statistics make it seem unlikely that the 4. In a nature reserve in India, people are total consumption of electricity determines its sometimes attacked by tigers. It is believed that cost to individual consumers. Recent increases the tigers will only attack people from behind. So in total consumption, especially during the hot for the past few years many workers in the summer months, have sometimes been reserve have started wearing masks depicting a accompanied by a decrease in the cost per unit human face on the back of their heads. While and at other times by an increase. many area residents remain skeptical, no worker wearing one of these masks has yet been Which of the following positions is best attacked by a tiger. supported by the information presented above? Which of the statements below, if true, would (A) It must be the case that the cost of best support the argument of those who electricity to consumers is what advocate the use of the mask? determines the total consumption. (B) Even though a correlation exists between (A) Many workers in the nature reserve who the total consumption of electricity and its do not wear the masks have been cost to consumers, no causal relation attacked recently by tigers. exists. (B) Workers in other nature reserves who (C) Further investigation into the way these wear similar masks have not been statistics were gathered is certainly called attacked recently by tigers. for. (C) No tigers have been spotted on the nature (D) The cost of electricity depends upon reserve in recent years. something other than the total (D) Many of the workers who wear the masks consumption of electricity. also sing while they work in order to (E) The cost-per-unit of electricity to frighten away any tigers in the area. consumers is dependent on the total (E) The tigers have often been observed electricity consumption. attacking small deer from in front rather than from behind. GO ON TO THE NEXT PAGE 198
  • 23. CRITICAL REASONING PRACTICE SETS 5. Since Arlene Hodges was installed as president 6. Although recently introduced with a wave of of the Caralis corporation, profits have increased publicity, combined audio/video receivers are by an average of 11 percent per year. During the proving tough to sell. As a result, the tenure of her predecessor, the corporation’s manufacturers keep advertising, and offering profits averaged a yearly increase of only 7 handsome discounts. What the manufacturers percent. Obviously Ms. Hodges’ aggressive have yet to see is that the public’s reluctance marketing efforts have caused the acceleration isn’t due to the price of the product; the public is in the growth of Caralis’ profits. still debating the even more fundamental question of --------. Which of the following, if true, would most weaken the conclusion drawn above? Which of the following best completes the passage above? (A) The corporation’s new manufacturing plant, constructed in the past year, has (A) whether the product is worth its price resulted in a 15 percent increase in (B) whether there is a use for such a product production capacity. (C) whether the models will soon become (B) For each year of Ms. Hodges’ presidency, obsolete the corporation’s financial records show an increase in profits over the previous (D) whether the uses for such a product year. outweigh its high cost (C) During the tenure of Ms. Hodges’ (E) whether more stylish models will appear in predecessor, the corporation began an the future advertising campaign aimed at capturing consumers between the ages of 24 and 35. (D) Since Ms. Hodges became president, the corporation has switched the primary focus of its advertising from print ads to radio and television commercials. (E) Just before he was replaced, Ms. Hodges’ predecessor directed the acquisition of a rival corporation, which has nearly doubled the corporation’s yearly revenues. GO ON TO THE NEXT PAGE 199
  • 24. P R A C T I C E S E T S A N D E X P L A N AT I O N S 7. Although statistics and definitions are inexact, Questions 8 and 9 are based on the following. educated guesses put the number of refugees In spite of what its critics say, the new, worldwide at well over 10 million. The expensive artificial kidney implant is a boon overwhelming majority prefer to return to their to modern medicine. These critics should native land than to emigrate to a foreign one. remember that the first heart transplants The millions of refugees from Afghanistan are stimulated an enormous amount of beneficial sufficient proof: Despite the toll the war and medical research, even though the transplant subsequent fighting have taken on their country, technique was later rejected in favor of less very few have applied for permission to invasive surgical procedures. emigrate. Which of the following, if true, would most strengthen the argument above? 8. The author defends the artificial kidney implant against the critics by (A) Most refugees are as reluctant to emigrate as are the refugees from (A) attacking her opponents’ method of Afghanistan. circular reasoning (B) The refugees from Afghanistan fled what (B) implying an analogy between the benefits they considered political oppression rather of the artificial kidney implant and the than economic disaster. benefits of the early heart transplants (C) Most of the children born to refugees (C) pointing out a contradiction implicit in their prefer to remain in their adoptive country criticism of the implant’s high cost rather than return to the land their parents (D) criticizing the professional objectivity of left. her opponents rather than their claim (D) Although refugees flee their homes for a (E) implying an analogy between the critics’ variety of different reasons, the opposition to the artificial kidney implant overwhelming majority are looking for and their opposition to the early heart improved living conditions. transplants (E) The number of refugees worldwide has risen dramatically over the last ten years. 9. Opponents of the artificial kidney implant could best defend their position against the author’s argument by pointing out that (A) once perfected, the new artificial kidney implant will cost very little (B) many of those who oppose the new artificial kidney implant have very good reputations in the scientific community (C) the high cost of the new artificial kidney implant does not necessarily mean that its creators are incompetent or profligate (D) the medical research that the artificial kidney implant will stimulate is similar to the research stimulated by the heart transplants (E) the fact that one medical innovation stimulated beneficial research does not mean that all such innovations will do so GO ON TO THE NEXT PAGE 200
  • 25. CRITICAL REASONING PRACTICE SETS 10. The population increase over a given year is 11. Chef: An ordinance ought to be passed banning always calculated as a percentage of the midtown street vendors from selling food previous year’s population, with a population within a certain proximity to restaurants. decrease being recorded as a negative increase. With their high rents and costs, In 1990 Essex County and Union County both restaurants cannot be expected to experienced a population increase of more than compete with the vendors. Even in cases 3 percent. In 1991 Essex County and Union where these vendors sell food completely County both experienced a population increase different from the restaurant’s bill of fare, of 1.5 percent. In 1992 Union County the price differential is enough to attract experienced a negative increase in population of to the vendors customers who would Ϫ1.7 percent. Essex County had 209,100 otherwise have eaten in the restaurants. residents in 1990 and 209,000 residents in 1992. Which of the following, if true, would most weaken the argument above? If the information above is accurate, which of the following must be true? (A) The food served in midtown restaurants is better than that sold by street vendors. (A) Both counties experienced positive (B) Most street vendors who sell food would population increases in the years 1990 suffer losses from being forced to move to and 1991, yet both experienced negative other locations. population increases in 1992. (C) Most customers who eat in midtown do so (B) In 1992 more people moved from Union on expense accounts, and do not pay for County to Essex County than moved from their own meals. Essex County to Union County. (D) There are not enough police officers to (C) In 1990 there were more people living in enforce regulations requiring street Union County than in Essex County, but in vendors to move a certain distance from 1992 there were more people living in restaurants. Essex County than in Union County. (E) The average profit on food sold by a street (D) Both counties experienced negative vendor is roughly the same percentage as population increases in the years 1990 that on food served in restaurants. and 1991, yet both experienced positive population increases in 1992. (E) Both counties experienced positive population increases in the years 1990 and 1991, yet while Essex County experienced a positive increase in 1992, Union County had a negative increase. GO ON TO THE NEXT PAGE 201
  • 26. P R A C T I C E S E T S A N D E X P L A N AT I O N S 12. It has been against the law for federal agencies 13. The manufacturer of DTF claims its product and federal contractors to discriminate against a reduces facial wrinkles and wishes to sell it as a qualified job applicant because of a disability. pharmaceutical. The Food and Drug Now that Congress has approved legislation to Administration (FDA), however, has ordered expand these existing provisions to cover private lengthy and costly experiments to determine industry as well, the number of disabled people whether DTF truly reduces facial wrinkles. The who are involuntarily unemployed will drop manufacturer, a small cosmetics firm, lacks the substantially. resources to carry out the required research and will probably comply with less rigorous FDA The author of the above argument must be regulations by labeling DTF a cosmetic. assuming which of the following? Which of the following can be most reasonably (A) Many congressmen were reluctant to pass inferred from the statements above? the new legislation to prevent discrimination against the disabled. (A) Cosmetics are among the products not (B) Some private employers in the past regulated by the FDA. deliberately chose not to hire qualified but (B) Only established pharmaceutical firms disabled job applicants. have the capital required to enter new (C) The federal government currently employs products in the market. more disabled people than does private (C) The makers of DTF thought they would sell industry. greater quantities of their product if it were (D) The approved legislation would stop a pharmaceutical rather than a cosmetic. discrimination against the disabled in the (D) The FDA regulates claims made about public and private sectors. pharmaceuticals more strictly than claims (E) Many disabled people voluntarily choose made about cosmetics. to remain unemployed. (E) DTF has very little effect, if any, on facial wrinkles and would not have received FDA approval. STOP! END OF TEST 202
  • 27. CRITICAL REASONING PRACTICE SETS 14. Homeowner: Recent drops in the value of our 15. Plant Y thrives in environments of great sunlight homes are due to the and very little moisture. Desert X is an undesirability of living near the environment with constant, powerful sunlight, recently opened 24-hour bus and almost no moisture. Although Plant Y thrives depot. in the areas surrounding Desert X, it does not exist naturally in the desert, nor does it survive Transportation official: The police, not the bus long when introduced there. depot, are at fault. Survey data shows that most prospective Which of the following, if true, would be most homeowners avoid the useful in explaining the apparent discrepancy community because of its high above? crime rate. (A) Desert X’s climate is far too harsh for the Which of the following, if true, would be the most animals that normally feed on Plant Y. effective rebuttal that the homeowner could (B) For one week in the fall, Desert X gets make to the argument put forward by the consistent rainfall. transportation official? (C) The environment around Desert X is (A) The community’s crime rate has risen ideally suited to the needs of Plant Y. nearly as quickly as the value of homes (D) Due to the lack of sufficient moisture, has fallen, over the same period of time. Desert X can support very little plant life. (B) Those homeowners whose homes are (E) Plant Y cannot survive in temperatures as situated in the immediate area of the bus high as those normally found in Desert X. depot must endure constant noise. (C) The constant activity and commerce generated by the bus depot has made the community a favored hangout for thieves and other criminals. (D) The community’s police force has not seen a pay raise or increase in manpower for the last five years. (E) The surveys of prospective homeowners did not include questions concerning the desirability of living in a community with a 24-hour bus depot. GO ON TO THE NEXT PAGE 203
  • 28. P R A C T I C E S E T S A N D E X P L A N AT I O N S 16. The World Automobile Association (WAA) publishes a list of the “Best and Worst Drivers of the World,” ranking the drivers of every nation according to the number of traffic deaths per mile driven in that country. Each of the following, if true, would by itself provide a logical objection to using the WAA’s ranking as a representation of the quality of drivers in each nation EXCEPT: (A) The roads in some countries are in bad repair and are therefore more dangerous than roads in other countries. (B) The average driver in industrialized countries can afford to maintain his or her car in better condition than can the average driver in less developed countries. (C) Some countries contain hundreds of thousands of miles of road while other countries contain relatively few miles of road. (D) Minor accidents that would cause little injury in many countries are often fatal when they occur in extremely mountainous countries. (E) Because of differences in national economies, the average car in some countries contains many more passengers than does the average car in other countries. STOP! END OF SET 204
  • 30. P R A C T I C E S E T S A N D E X P L A N AT I O N S ANSWER KEY 1. D 2. A 3. D 4. A 5. E 6. B 7. A 8. B 9. E 10. A 11. C 12. B 13. D 14. C 15. E 16. C 206
  • 31. CRITICAL REASONING EXPLANATIONS CHAPTER 21 CRITICAL REASONING EXPLANATIONS SET 1 Answer Key: 1. C 7. C 13. D 2. D 8. E 14. B 3. C 9. E 15. B 4. A 10. A 16. A 5. B 11. E 6. A 12. D 1. C background. Supervisory experience (B) isn’t the The Conclusion: Union reform movements should same as coming up through the ranks. Labor unions build staffs out of workers who have come up the having problems, which the author admits, isn’t the ranks. same as (D) most of them unfairly representing workers’ interests. That’s an overstatement. The Evidence: Union movements are currently “Democratic” (E) is a new term, and one the suffering from a problem: Their staffs consist of argument doesn’t need. college-educated professional types who don’t understand the concerns of the worker. 2. D If the author believes that hiring up-from-the-ranks The Conclusion: None is stated, but as the question workers (an idea introduced in the conclusion) will stem alerts us, one is implied. cure that problem, he must be assuming that these former workers do understand workers’ real The Evidence: A plant opened in Country Y during a concerns. revolution ten years ago has always generated substantial profits. There’s no need to assume that higher education lessens people’s identification with their class The author draws an analogy between the two plants. background (A), since the author hasn’t said that the since the one in Country Y has made money, so too, lawyers, economists, and experts who don’t she implies, will the one in Country X. understand workers come from a working class The author’s point is that the plant can be successful 295
  • 32. P R A C T I C E S E T S A N D E X P L A N AT I O N S despite the war, not that (A) the plant will be The implication is that Zydol is more cost-effective successful because of the war. The author presents the than aspirin: you get a much better bargain with a two countries as similar, so she’s not arguing that (B) bottle of Zydol. But if the aspirin bottle contains one is more stable than the other. The author is more than twice as many tablets as the Zydol bottle, attacking her opponents’ argument, but not (C) their then it’s the aspirin bottle that gives you more pain motives. She makes no judgment (E) as to which reliever for your money. plant will be more successful. Remember, comparing and contrasting things that are considered equivalent The argument concerns which is the better bargain, in the stimulus is a common wrong answer type for not which is the better pain reliever. Thus it doesn’t inference questions. matter (B) that the ingredient is the same. (C) points to some consumers who won’t want Zydol. Perhaps, but one, these may be very few, and two, this doesn’t 3. C attack the author’s reasoning, which is based on cost. The Conclusion: There is none stated. One is implied, Pain sufferers who don’t have a doctor’s prescription though, and we’re to locate it among the answer are left with the same choice between Zydol and choices. aspirin, so (D) doesn’t weaken Zydol’s case against The Evidence: Since deregulation, low air fares have aspirin. Even if you make an unwarranted leap and correlated with increased miles flown. Recently prices assume that there may be fewer of the large Zydol have risen, but the number of passenger-miles flown pills (E) per bottle, choice (A) clearly does a better job is still increasing. of arguing along this line. The passage begins with a correlation and then 5. B breaks it in the next sentence. What can you infer The Undergraduates’ Conclusion: If grades are from this? Not very much, you may have thought, assigned by graduate assistants, regular full-time fac- and you’d be right: the key to this question is not ulty will devote less time to undergraduate work. drawing an unwarranted sweeping conclusion from the evidence. What you can safely conclude is that the Their Evidence: There really isn’t any: just the idea air fare doesn’t, by itself, determine the amount of air that giving grading responsibilities to graduate travel. students will somehow move regular full-time faculty away from the undergraduates’ work. That air-fares don’t have any effect on air travel (A) is too strong. We can’t infer why passenger-miles flown The only thing to do is go through the choices to see have increased (B); it might be at the expense of other which question we can answer. Since the forms of transportation, but it might not be. The undergraduates are against the proposal, it’s safe to author merely states that the air fares have risen conclude that they want the full-time faculty to stay recently, but never mentions a cause (D) like interested in their work. We have no information “takeovers.” So far, the recent rise in air fares hasn’t about the fairness of graduate assistants’ grades (A). reduced passenger-miles flown, but that doesn’t All we know about the discussion sections (C) is that mean (E) that a further increase won’t. graduate assistants now hold them; we aren’t told whether or not full-time faculty may hold them. (D) asks about the validity of the faculty’s grades. We 4. A know that some people have proposed that graduate The Conclusion: Consumers can be expected to assistants be given some grading responsibility, but switch to Zydol. that doesn’t imply (D) that they believe the faculty’s grades have been made less valid by the graduate The Evidence: Although one has to take two aspirins students’ “contact” with undergraduates. And we’re to get the relief provided by one Zydol, nevertheless a told absolutely nothing about (E) the faculty’s bottle of Zydol costs the same as a bottle of aspirin. opinion of the proposal. 296
  • 33. C R I T I C A L R E A S O N I N G E X P L A N AT I O N S 6. A some less. The reasons for the growth difference The Conclusion: Air travel is becoming more danger- between boys and girls (D) isn’t relevant to the ous. growth differences between coed and single-sex schools. Since the study is only interested in how The Evidence: In the last year there have been seven much students grow during the first two years of collisions and over 700 deaths, the highest number of high-school, it’s irrelevant how tall they are to start deaths ever. with (E). We want something that suggests that the increase in fatalities doesn’t prove an increase in danger. If more 8. E and more people are traveling by airplane, an increase The Conclusion: Exercise is good for the mind as well in the number of deaths doesn’t prove that air travel as for the body. is becoming more dangerous. The question of how dangerous air travel is can’t be answered unless we The Evidence: Exercise increases the speed at which know the proportion, not the number, of passengers blood circulates, allowing the brain to receive more who get killed. oxygen and thus concentrate better. The author’s contention isn’t undermined by (B) the The main point is the conclusion, that exercise is reasons for the lack of safety. Pointing out more good for the mind as well as the body. More generally, dangerous methods of transportation (C) doesn’t exercise serves more than one purpose. deny that the danger of air travel is increasing. (A) goes out on a limb; the author’s point is the Likewise, the fact that other forms of transportation beneficial effects of exercise, not the unlimited are also getting more dangerous (D) doesn’t dent the benefits of oxygen. While exercise aids mental author’s claim. Fewer passengers per place (E) means activity, it’s not (B) a mental activity as such. Whether more collisions, but we already know how many exercise aids the brain more or less than he rest of the collisions there were. body (C) isn’t discussed. This is a classic wrong answer type: the choice that compares two items the 7. C stimulus treats equally. While the author argues that The Conclusion: Attending a single-sex high school exercise is beneficial to the mind, “greatly improving” promotes growth. concentration (D) is an overstatement. The Evidence: During the first two years, the average student in an all—boys school grew five inches and 9. E the average girl in an all—girls school grew four Ernesto’s Conclusion: VCR sales will decline. inches. Ernesto’s Evidence: The saturation level for The question stem tells you that something has been households has been reached. left out of the argument. The conclusion compares Milton’s Conclusion: VCR sales won’t decline. single-sex schools to coed schools, but the evidence only cites data from single-sex schools. Until we Milton’s Evidence: More and more films are released know how much the average student grows in co- on videotape. educational schools, we have no evidence that the growth in single-sex schools is greater. Milton simply ignores Ernesto’s argument that the saturation point has been reached for VCR’s. He Even if we knew the answer to (A) we’d still have no concentrates on the new cassettes coming out, but the evidence from the coed schools. (B) is a rather silly availability of new cassettes doesn’t mean people question: we’re dealing with average growth, so we’d need buy or will buy more recorders. Milton ignores expect some boys to grow more than the average and the relevant issue of saturation raised by Ernesto, and 297
  • 34. P R A C T I C E S E T S A N D E X P L A N AT I O N S simply assumes that sales of VCR’s will continue as 11. E before. The Conclusion: Introducing a new cereal will only hurt the brands Coolidge already produces, so Milton’s evidence doesn’t disprove Ernesto’s evidence Coolidge shouldn’t introduce another brand of (A). The videos appearing each year have nothing to cereal. do with whether the saturation point in VCR ownership has been reached. “Finding a gap” (B) in The Evidence: The introduction of a new brand of an argument means demonstrating that the evidence cereal doesn’t increase the total number of cereal doesn’t lead to the conclusion; Milton merely brings buyers, but only encourages those who already buy up some irrelevant information that has nothing to cereal to switch brands. do with Ernesto’s reasoning. Milton does cite an issue that Ernesto ignored (C), but the claim that this issue There’s an assumption here: that the new brand will “outweighs” the issue of saturation is unsupported. only attract those who currently buy other brands of Milton does fail to speak to Ernesto’s point (D) but Coolidge’s cereal. If the new brand steals buyers from not for the bizarre reason given here. Ernesto has competitors’ cereals, then it will help Coolidge by absolutely nothing to say about “other leisure-related adding to its total sales. products” there’s no reason for Milton to raise this Even if the population does increase and total cereal issue. sales with it (A), a new brand might still hurt the sales of Coolidge’s established cereals. The new cereal may 10. A sell well (B), but if that just means it’s stealing lots of The Conclusion: People must already have been using buyers from other Coolidge cereals, what good is it? Type C weapons in this area by 2500 B.C. The fact that Coolidge has only a few brands (C) doesn’t make it likely that a new cereal brand would The Evidence: Depictions of the weapons exist on the steal buyers from competitors’ brands rather than walls of a cave that was sealed off in 2500 B.C. from other Coolidge brands. Neither does the fact We need to show that the paintings either don’t that some brand switching is usual even when a new depict weapons that were in use or that the paintings brand isn’t introduced (D). were made after 2500 B.C. The archaeologists are assuming that there was only one entrance to the 12. D cave. If there’s another entrance to the cave that was The Conclusion: Not eating cheese protects children only sealed much later, then people could have from childhood diseases. entered the cave and made the paintings long after the first entrance was sealed. The Evidence: All the children at summer school who have never had measles or chicken pox have also If anything, the existence of the weapons in India (B) never eaten the cheese served in the school lunches. might be considered to strengthen the argument, since it shows Type C weapons were in existence as The author deduces a causal relationship: not eating early as 2500 B.C. Pushing the date of the eruption cheese leads to protection from childhood diseases. I back (C) also strengthens the argument by making it hope you saw that this has numerous holes. Four of likely that the paintings were done even earlier than the choices help to fill those holes; the fifth does not. claimed. Without some evidence to back it up, the That fifth choice is (D). If research shows that opinion of the experts (D) isn’t worth much. That abstaining from cheese products is a major cause of identification was difficult (E) is not the same as its some childhood diseases, then the health official’s being uncertain or controversial. claim that children can protect themselves from disease by not eating cheese is flat-out wrong. 298
  • 35. C R I T I C A L R E A S O N I N G E X P L A N AT I O N S It’s important that the author be able (A) to infer Her Evidence: Although the same plant can produce from measles and chicken pox (in the evidence) to the same amount of electricity in either site, building other childhood diseases. Also necessary is that the at Abaco will cost twice as much money. observations be accurate (B) and the students’ behavior at home mirror their behavior at school (C). Looking at the question stem first here really helps Each of those speaks to the legitimacy of the you narrow your focus when reading the stimulus. correlation. Most important, though, is that there The only thing you’re interested in is why the Abaco really be (E) a causal connection, rather than just a site will cost more than twice as much as the Bornos correlation, between cheese eating and childhood site. The stimulus doesn’t give you any hint as to how illness. the sites are different, so you’ll have to rely on the correct answer to provide a complete explanation by itself. If the Abaco site is in the middle of nowhere, 13. D and requires constructing new roads and importing The Consulting Firm’s Conclusion: The employees’ laborers, then of course it’s going to be more dissatisfaction is all in their heads. expensive to build there. Its Evidence: 75 percent of the employees are The benefits of the Bornos plant to farms (A) don’t dissatisfied. An investigation showed no explain its lower cost of construction. Neither do the uncommonly bad working conditions. benefits to nearby city residents (C). Possible We need the one choice that doesn’t weaken the environmental damage caused by a plant built at consulting firm’s conclusion; that’s a good tip that the Abaco (D) is another reason for building at Bornos, conclusion isn’t solidly based. The mere fact that the but it’s a different reason. The problem or issue of complaints vary doesn’t hurt the firm’s conclusion expansion (E), like the environment, might merit that the complaints are based on psychological consideration, but it’s not a factor in determining the problems. In fact, it might even strengthen the firm’s cost of building. argument: every employee has his or her own mental hang-up, so every employee comes up with his or her 15. B own irrational complaint. The Conclusion: These five people can serve as mod- els for the type of person who can be helped by hyp- If many employees had been working at a different nosis. company’s facilities (A), then it could be these facilities, not mental problems, that are responsible The Evidence: A study showed that these five for the complaints. If the situation in Company P is previously disturbed hypnosis subjects had stable fairly normal (B), there’s no need for the firm to emotional conditions. hypothesize an unusual incidence of psychological problems among the workforce. If the firm’s The survey only found that at the time of the study definition of uncommonly bad conditions differs the five seemed to be doing OK. Remember, these from that of the workers (C), then the firm may well people were originally suffering from mood swings; have overlooked the real causes of employee maybe the study just caught them on a good day. If dissatisfaction. And if a battery of tests (E), showed that’s the case—if since 1988 these people have been no significant psychological problems, then the very experiencing dramatic mood swings and occasional cause cited by the firm is attacked. periods of health—then hypnosis hasn’t really helped them and they’re not good models. 14. B The author presented the people as models of The Director’s Conclusion: We should build the dam different types of people who can be helped, not as a at the Bornos site. single model of a single personality type, so they needn’t be similar (A). It doesn’t matter that the other 299
  • 36. P R A C T I C E S E T S A N D E X P L A N AT I O N S 45 people who underwent hypnosis didn’t get better (C); the argument is based on and concerns only the five who were stable. (E) fails to weaken the argument for the same reason. The concern of many psychologists (D) is well outside the scope. We need a statement that speaks about hypnosis and these five subjects. 16. A The New Theory’s Conclusion: Heat (from the burn- ing coal) causes columnar growth in junipers near burning underground coal veins. The New Theory’s Evidence: None really, except the correlation of columnar growth with these areas with underground fires. The Old Theory’s Conclusion: The abundance of carbon monoxide causes columnar growth. The Old Theory’s Evidence: None really, except the correlation of columnar growth with these areas with high carbon monoxide. When you scan the choices, you see that each presents a case of the cause with or without the effect or the effect with or without the cause. Since the two theories are in opposition, weakening the old theory is a way of strengthening the new one. Bearing in mind from the lesson the key issues in a causal argument, we recognize that a case of columnar growth where the cause claimed by the new theory (heat) is present, but the cause claimed by the old theory (carbon monoxide) is absent strengthens the new theory at the expense of the old. A case (B) where we get the new theory’s alleged cause (intense heat), without the alleged effect (columnar junipers) is of no help at all. A columnar juniper in an atmosphere with high carbon monoxide but no extra heat (C) strengthens the old theory. A case with both alleged causes without the expected effect (D) weakens both theories. Likewise columnar growth in the presence of both causes (E) does nothing to promote one theory over the other. 300